0% found this document useful (0 votes)
43 views59 pages

CAT Mock Test - 2

The document discusses the significance of Indian mythological stories, particularly the 'puranas', in teaching moral values and the concept of 'dharma'. It contrasts two interpretations of dharma: the verb-centric view emphasizing ritual and action, and the noun-centric view focusing on abstract ideals. Additionally, it explores the complexities of human irrationality and the philosophical discourse surrounding it.
Copyright
© © All Rights Reserved
We take content rights seriously. If you suspect this is your content, claim it here.
Available Formats
Download as PDF, TXT or read online on Scribd
0% found this document useful (0 votes)
43 views59 pages

CAT Mock Test - 2

The document discusses the significance of Indian mythological stories, particularly the 'puranas', in teaching moral values and the concept of 'dharma'. It contrasts two interpretations of dharma: the verb-centric view emphasizing ritual and action, and the noun-centric view focusing on abstract ideals. Additionally, it explores the complexities of human irrationality and the philosophical discourse surrounding it.
Copyright
© © All Rights Reserved
We take content rights seriously. If you suspect this is your content, claim it here.
Available Formats
Download as PDF, TXT or read online on Scribd
You are on page 1/ 59

MBA

CAT Mock Test - 2

Directions (1-4) Read the following passage and just simple moralising: by emulating the actions
answer the given questions. they prescribe, the stories help cleanse the mind,
As a child, for every summer vacation, my like a previously used but unwashed utensil, and
parents took me to Kerala in southern India to help improve our abilities to choose better in life.
spend three months with my aunt in her large However, nothing is preordained in this
family-estate. It was an age before televisions pedagogic nudge. Implicit in this model of
were widely available and therefore at night-time human birth, or more accurately the emergence
she told us stories from the vast oeuvre of Indian of sentient beings, are two outcomes: one can
mythologies called the ‘puranas’. These stories very well ignore these stories and ideals of self-
often involved a moral exemplar as a protagonist improvement and go on to accumulate more
– a hero who embodied Immanuel Kant’s ‘karmic’ muck, or one can live as per these stories
‘categorical imperative’ – whose life’s arc and steadily avoid ‘karmic’ burdens. The freedom
exemplified devotion, truth, sacrifice, love and to choose is yours. Amid all this, there remains a
other ennobling ideas. The dramatic twists in supervening question often left unanswered:
these stories came from the gods and their what is this dharma that these stories speak of
caprice, which tested the commitments of and point towards?
righteous men and women in the face of [...]‘Dharma’ was the froth born from performing
opportunities to abandon their ideals and save ritual. This efflorescence of meanings was vividly
themselves. They invariably never did, at great visible when the German linguist Karl Friedrich
cost to themselves. My aunt referred to these Geldner translated the Rigveda in 1928 and used
men and women as ‘symbols of dharma’ (dharma ‘no fewer than 20 renderings’ – from ‘law’, ‘order’,
prateekam), although she left that capacious ‘duty’ ‘custom’, ‘quality’ and so on – to describe
word – dharma – unexplained. dharma in various contexts.
[...]The locus of virtue in those stories lay not in In a recent conversation with Bibek Debroy,
ritual or rank but in sustaining commitment to an translator of the epic 10-volume Penguin edition
ideal that demanded some form of sacrifice. of the Mahabharata (2015), he told me that: ‘The
When faced with morally complex situations in basis of Sanskrit is the verbal root, the dhatu …
their own lives – be they a child listening to his Looking at it from the point of view of the root,
aunt, or an audience hearing them from a the verbal root – actually [‘dharma’] is very, very
religious teacher – the listeners could precise.’
theoretically ask themselves: ‘What would In this seemingly simple remark, however, lie the
Shravana the filial son do?’ Or: ‘What would seeds of two distinct types of Indian thought,
Savitri the faithful wife do?’ which began from the Rigveda and evolved over
[...]But beyond such instances of moral emulation, time: a ‘verb-centric’ understanding and a ‘noun-
at the heart of this pedagogic agenda was also a centric’ interpretation. As Daya Krishna put it in
model of how humans are born: they emerge in a 1997, the verb-centric view spawned the long-
new womb carrying along with them all the lived Mimamsaka (hermeneutic) traditions that
deeds and knowledge from their previous lives. lay a premium on verbal declarations, injunctions,
The role of these stories is therefore more than the ritual sacrifice, and ultimately a theology of

Android App | iOS App | PW Website


MBA

action (karma) – for which ‘dharma’ was the froth The stories provide a framework for
born from performing ritual. In contrast, a noun- moral cleansing, allowing listeners to
centric view grew into the Upanishadic tradition consciously decide their path between
that understood reality as being described by adhering to dharma and accumulating
‘being’ rather than by ‘becoming’. This view ‘karmic’ muck.
elevated non-ritualistic speculations and (D) The depiction of dharma in the stories is
methods of debate and, importantly, drew a link a model for self-improvement through
between reality and knowledge (jnaana). Thus, conscious effort, emphasising that
whereas for the Mimamsaka traditions, ‘dharma’ adherence to these ideals is a choice
is tied to instruction and performance of action, rather than a predestined path
for the Upanishadic worldview, dharma in its
Q3 Which of the following sets of keywords
essence is born out of a transcendent ideal: truth.
most accurately reflects what the passage
Q1 What was the purpose of the author is all about?
behind using the example of Karl Friedrich (A) Puranas, dharma, moral exemplars, self-
Geldner in the passage? improvement, ritualistic sacrifice, karmic
burden, Mimamsaka, Upanishadic
(A) To demonstrate the evolution of Indian tradition, philosophical foundations,
thought from a verb-centric to a noun- linguistic analysis
centric understanding of ‘dharma’. (B) Moral education, linguistic analysis,
(B) To illustrate the complexity and cultural practices, ethical dilemmas,
multiplicity of meanings associated with mythological stories, historical context,
the term ‘dharma’ in different contexts. ritual symbolism, Mimamsaka,
(C) To exemplify how the translation of Upanishadic tradition
ancient texts about dharma can reflect (C) Dharma, ritualistic practices, moral
both the Mimamsaka and Upanishadic stories, philosophical debate, karmic
traditions. consequences, ethical frameworks,
(D) To emphasize the significance of cultural traditions, pedagogic value,
linguistic precision in understanding the linguistic analysis
verbal root ‘dhatu’ in Sanskrit. (D) Ethical conduct, puranas influence,
dharma meanings, historical evolution,
Q2 The author uses the sentence "However,
moral reasoning, ritualistic practices,
nothing is preordained in this pedagogic
societal norms, philosophical
nudge" to imply:
foundations symbols of dharma
(A) Individuals have the autonomy to
choose whether to follow the moral Q4 Which of the following options best
lessons from the stories or not, describes the distinction between verb-
reflecting the freedom to either centric and noun-centric interpretations of
accumulate or avoid karmic burdens. dharma?
(B) The concept of rebirth involves carrying (A) The verb-centric interpretation views
past deeds and knowledge, yet dharma primarily through its ritualistic
individuals are not bound by these and and performative aspects, often linked
can exercise their free will in every new to societal norms, whereas the noun-
life cycle. centric interpretation treats dharma as a
(C)

Android App | iOS App | PW Website


MBA

set of unchanging, prescriptive laws that employing – in particular, where their notion of
govern behavior. (ir)rationality is substantive or structural.
(B) The verb-centric view considers dharma If we’re employing a substantive notion of
as a set of moral principles derived from rationality, it’s hard to contest that human beings
historical rituals, while the noun-centric are very often irrational. We often have beliefs
view focuses on dharma as a that are not well supported by evidence – the
philosophical concept with a fixed, climate-change deniers, flat-Earthers and fairy-
universal meaning across all contexts. believers just discussed are cases in point. And
(C) The verb-centric interpretation of we often do things that there’s very strong
dharma emphasizes ritualistic actions reason not to do, as when we engage in
and the performance of duties as behaviours very harmful to our health, lash out at
outlined in the Mimamsaka traditions, others intemperately, or fall victim to scams and
while the noun-centric interpretation hucksters.
focuses on abstract, non-ritualistic ideals A more difficult question is how widespread
of truth and reality as emphasized by structural irrationality – incoherence – is. It’s not
the Upanishadic tradition. unusual to hear it said that we’re very incoherent
(or, more colloquially, inconsistent). But, on
(D) The verb-centric interpretation sees reflection, there’s a way in which paradigm cases
dharma as a flexible and evolving of incoherence are not just irrational but
concept tied to personal morality and borderline unintelligible. Return to the shoes/mall
ethical conduct, while the noun-centric example, and my hypothetical pronouncement: ‘I
interpretation rigidly defines dharma in acknowledge that, in order to get the shoes, I
terms of ancient rituals and religious have to go to the mall. And I do intend to get the
duties. shoes. But I have no intention whatsoever to go
to the mall!’ If I cheerily announced this to you,
Directions (5-8) Read the following passage
you might not just think that I’m irrational, but
and answer the given questions.
struggle to even make sense of how I could really
It’s a truism of contemporary middlebrow
be in the state of mind that I claim to be in. (Am I
discourse that human beings are deeply
joking?) You might think: ‘If he knows that the
irrational. According to a simplified but not
only way to get the shoes is to go to the mall,
entirely fabricated popular narrative, classical
and he doesn’t intend to go to the mall, then that
economists naively assumed that individuals are
shows that he doesn’t intend to get the shoes
perfectly rational, before contemporary
after all. Maybe he’d like to have the shoes, but if
psychology (and ‘behavioural economics’) came
he really intended to get the shoes, he’d plan on
along to burst their bubble by experimentally
going to the mall.’
demonstrating the depths of human irrationality.
I think this reaction is on to something. In more
Irrationality-talk in public discourse got another
general terms, the thought is this: it’s part of
boost from the advent of Trumpist politics, with
what it is to intend to do something that, when
academic books and newspaper op-eds
you believe that doing some second thing is
frequently using the word in characterising
necessary for doing the first thing, you’ll at least
Trump and his devotees. It’s striking, though, that
tend to come to intend to do the second thing as
many of the loudest voices professing the
well. If you don’t have this tendency, then you
magnitude of human irrationality tend not to say
don’t really count as intending to do the first
what understanding of irrationality they’re
thing after all. And that puts limits on the extent

Android App | iOS App | PW Website


MBA

to which your intentions can really be incoherent. (B) Examining structural rationality can shed
Similar points hold in the most egregious cases of light on seemingly irrational behavior.
incoherent beliefs. If someone announces: ‘I (C) The concept of "irrationality" is more
believe that great cooks never overcook eggs, useful in political rhetoric than
and I believe that I am a great cook, but I believe philosophical discourse.
that I have overcooked the eggs,’ the most (D) Substantive rationality is a more realistic
natural way to hear this is as a joke. If they really measure of human decision-making
do believe they’ve overcooked the eggs, they than structural rationality.
either don’t really believe that great cooks never
Q7 The author's discussion of "Jack" who
overcook eggs, or don’t really believe they’re a
believes both that it is raining and that it is
genuinely great cook. Or so it’s very tempting to
not raining is meant to:
say.
(A) Demonstrate the difficulty of imagining
Philosophers writing about irrationality –
someone who could hold such a belief,
including, on occasion, myself – sometimes
suggesting that such incoherence is
overlook this, breezily describing cases of
beyond our comprehension.
outrageous incoherence as if there’s nothing
(B) Criticize philosophers for often relying
puzzling about them. They write things like:
on unrealistic scenarios in their
‘Imagine Jack, who believes that it’s raining and
discussions of irrationality, neglecting to
also believes that it’s not raining.’ If we slow
consider the practical implications of
down and take a reality check, though, it’s not
their theories.
clear how to go about imagining Jack. And that’s
(C) Highlight the absurdity of holding
because it’s not clear what kind of state of mind
contradictory beliefs, even if they seem
Jack could be in that would make it correct to
logically consistent on the surface.
describe him both as believing that it’s raining
(D) Question the validity of the concept of
and believing that it’s not raining. Normally, we
"irrationality" itself, suggesting that it is
think that part of what it is to believe that it’s
a meaningless category when applied to
raining is to deny claims like ‘it’s not raining’, and
human behavior.
vice versa. Many philosophers also think that
believing something involves tending to act as if Q8 It can be implied from the passage that the
it’s the case. But what would it be to act as if it’s popular notion of human irrationality often
raining and as if it’s not raining? falls short because:
(A) It conflates true irrationality with simply
Q5 The author suggests that the tendency to
having beliefs or engaging in actions
dismiss blatant incoherence as jokes stems
that others may disagree with or find
from our belief in the connection between:
undesirable.
(A) Beliefs and desires
(B) It oversimplifies the concept of
(B) Intentions and actions
rationality, neglecting to consider the
(C) Desires and social expectations
nuanced ways in which humans can be
(D) Intentions and future predictions
both rational and irrational in different
Q6 Which of the following statements would contexts.
the author MOST AGREE with? (C) It overlooks the fundamental fact that
(A) People with contradictory beliefs are human beings are inherently irrational
simply confused and need better creatures, and any attempt to explain
information

Android App | iOS App | PW Website


MBA

their behavior in terms of rationality is Only with Frege [ie analytic philosophy] was the
ultimately futile. proper object of philosophy finally established:
(D) It relies on a superficial understanding namely, first, that the goal of philosophy is the
of the concept of "irrationality," failing to analysis of the structure of thought; secondly,
differentiate between substantive and that the study of thought is to be sharply
structural irrationality. distinguished from the study of the
psychological process of thinking; and, finally,
Directions (9-12) Read the following passage
that the only proper method for analysing
and answer the given questions.
thought consists in the analysis of language.
For a long time, I had been enduring a crisis of
Bliss it must have been in that dawn to be alive!
meaning. I wanted to live a life that mattered, to
But the French Revolution went from equality to
do things that were valuable – and I was
tyranny, and in time, it turned out that Dummett
increasingly haunted by the suspicion that
had been too optimistic about analytic
nothing really mattered, that everything was
philosophy. The programme was revised and
ultimately meaningless. I decided that my new
ultimately abandoned.
research project would be on the meaning of life.
But the term ‘analytic philosophy’ has outlasted
I worked in a tradition of philosophy that people
the historical movements of analytic philosophy.
still call ‘analytic’. The basic idea of analytic
While it remains usual to speak of analytic
philosophy when it was first propagated was
philosophy and analytic philosophers, nobody
simple. At its core, it consisted of G E Moore’s
nowadays can say what it really means.
favourite question. Someone would say
Some people associate it with clarity, which is
something like: ‘Being is indivisible’, and Moore
hilarious if you actually read analytic philosophy.
would ask, ‘But what on earth does that mean?’
Others say it has something to do with ‘rigour’.
To put this in more theoretical terms, the big idea
This may be closer to the truth, but only if you
behind analytic philosophy was to replace
take it as something to do with rigor mortis.
metaphysics with linguistic analysis. Advocates of
Consider Susan Wolf writing on meaning in life.
this ‘linguistic turn’ believed, in Richard Rorty’s
She has just expressed the idea that the ‘best
words, that ‘philosophical problems are problems
sort of life is one that is involved in, or
which may be solved (or dissolved) either by
contributes to something “larger than oneself”.’
reforming language, or by understanding more
But as soon as Wolf has said this, she realises she
about the language we presently use.’ The way to
has not been rigorous, that the thought has not
make progress on the question of God’s
been properly explained. She immediately tells
existence was not to find more arguments for
us that: ‘[c]ontemplation of the case of Sisyphus
and against Her existence. Rather, one made
should, however, be enough to show that this
progress by investigating what it meant to say
“larger” must be understood metaphorically. We
‘God exists’.
may, after all, imagine the rock Sisyphus is
Today, it’s hard to fully inhabit the excitement felt
endlessly pushing uphill to be very large.’
by the pioneers of analytic philosophy and their
I think we can all agree that this is very rigorous.
immediate descendants, but it’s impossible to
The thought has been pursued until there is no
doubt that there was considerable excitement at
more thought possible. The lemon has been
the time. Michael Dummett, a Wykeham
squeezed dry. Sisyphus could have been pushing
Professor of Logic at the University of Oxford,
a very large rock up that hill. The largeness of
and not therefore a man given to emotional
rocks, we now see, is not the type of largeness
pronouncements, claimed that:
that Wolf had in mind. It is a different type of

Android App | iOS App | PW Website


MBA

largeness. One might almost venture to call it… repetitive and ultimately futile.
metaphorical. (C) Sisyphus's life would be evaluated
Another way people have tried picking out based on the metaphorical significance
analytic philosophy is to base it on a geo- of the rock he pushes.
linguistic criterion and call it Anglophone (D) Sisyphus's life would be considered
philosophy. But this is very unfair to the poor meaningful only if the rock he pushes is
German professors churning out pages of turgid large enough to symbolize a greater
prose in what Bernard Williams called the ‘style purpose.
[that] tries to remove in advance every
Q11 The sentiment "Bliss it must have been in
conceivable misunderstanding or
that dawn to be alive!" is used to:
misinterpretation or objection, including those
(A) Praise the intellectual achievements of
that would occur only to the malicious or the
early analytic philosophers.
clinically literal-minded.’
(B) Ironize the overconfidence of the
Q9 The author's initial statement, "I wanted to movement's founders.
live a life that mattered, to do things that (C) Emphasize the importance of clear
were valuable – and I was increasingly goals in philosophy.
haunted by the suspicion that nothing really (D) Contrast the initial excitement with the
mattered, that everything was ultimately later disillusionment.
meaningless," suggests:
Q12 The final paragraph on "Anglophone
(A) A deep existential crisis fueled by the
philosophy" implies that the author:
author's desire to find meaning in an
(A) Supports the use of a geo-linguistic
indifferent universe.
criterion for categorizing philosophies.
(B) A yearning for a life of significance, a
(B) Finds the writing style of German
quest for personal fulfillment in a world
philosophers cumbersome.
that often seems devoid of inherent
(C) Believes geographic origin shouldn't
value.
define philosophical approaches.
(C) A growing awareness of the futility of
(D) Prefers the clarity of Anglophone
human endeavors and the transient
philosophy to German styles.
nature of existence.
(D) A philosophical dilemma that the author Directions (13-16) Read the following passage
sought to resolve through rigorous and answer the given questions.
intellectual inquiry. Read the given passages carefully and answer
the questions given below each passage.
Q10 If the author were to accept Susan Wolf's
definition of a meaningful life as one that
Manifestly political novels have always aroused
is "involved in, or contributes to something
some degree of suspicion. Orwell famously
'larger than oneself,'" how would the
categorized Harriet Beecher Stowe's anti-slavery
author's view of Sisyphus's life change?
novel Uncle Tom's Cabin (1852) as a “good bad
(A) Sisyphus's life would be considered
book” - crude yet effective – and Milan Kundera
meaningful because he is constantly
in turn dismissed Orwell's Nineteen Eighty-Four
engaged in a task that transcends his
(1949) as “political thought disguised as a novel”.
individual existence.
In a devastating review, Whittaker Chambers said
(B) Sisyphus's life would be deemed
that Ayn Rand's colossal philosophical tract Atlas
meaningless because his actions are

Android App | iOS App | PW Website


MBA

Shrugged (1957) “can be called a novel only journalist Edward Bellamy produced a colossal
devaluing the term... Its story merely serves Miss bestseller that was said to have introduced more
Rand to get customers inside the tent, and as a Americans to socialism than Karl Marx. The
soapbox for delivering her Message.” book's influence extended to suffragists,
populists, HG Wells, Morris, Franklin D Roosevelt
Resistance to a message has only grown stronger and Clement Attlee, who told Bellamy's son Paul
since then. It seems that the tides of literary that their postwar Labor government was “a child
fashion have left the campaigning novel high and of the Bellamy's ideal.” Bellamy did not transform
dry. “We've all drunk in the idea that you don't the world to the extent that he had hoped, but
reward a book for its subject matter but its he certainly left it somewhat different.
literary craft,” Leith says. “while the consistency
Q13 Which of the following best describes the
of the metaphors is fair game, the value of their
relation between the first and the last
ideas is less so.” This critical focus on form and
paragraphs?
language rewards a book as unusual as Milkman
(A) The former presents a literary trend,
but creates a frosty climate for any author with a
while the latter gives an extension of
blatant reformist agenda.
the trend.
(B) The former implies a generalization,
“When you study literature, you learn the mantra
while the latter gives an exception to
that great art is not political,” agrees Joanna
that generalization.
Kavenna, whose latest novel Zed is a wry satire
(C) The former implies a generalization
about the unravelling of a supranational tech
about a literary genre, while the latter
giant. “you read that ideas are somehow an
talks about another genre.
imposition on fiction. But as soon as you create a
(D) The former introduces a literary genre,
character or write a story, you're into an idea
while the latter summarizes the main
about society whether you like it not.” Influenced
argument of the passage.
by Kafka, Gogol and the “tradition of unease”,
Kavenna prefers to deploy irony to express the Q14 It can be inferred that Joanna Kavenna
strangeness of modern life. More didactic fiction, and William Morris are likely to differ in
she says, is prone to being dismissed as which of the following?
propaganda. “Writers like [William] Morris could (A) While Joanna uses satire and irony as
be very certain: this is my political program, I tools of her fiction, William Morris did
want these outcomes. They were so sure of not use these literary tools in his
themselves and where they were in society. Our fiction.
world is much more volatile and the audiences (B) While Joanna wrote fiction related to
are much more sophisticated. People are so wary technology, William Morris wrote books
of blunt polemic – quite rightly – that we only about politics.
question the motives of everything all the time.” (C) Expression of political views is
necessary for Joanna to create fiction,
It wasn't always the case that readers recoiled while it is the purpose for William
from proselytizing fiction. Take for example, Morris.
Looking Backward: 2000- 1887 (1888). Wrapping (D) While Joanna's professional success is
his prospectus for American socialism inside the limited, William Morris' professional
tale of a Bostonian aristocrat who sleeps for 113 success is exemplary
years and awakes in an egalitarian utopia, the

Android App | iOS App | PW Website


MBA

Q15 The passage supplies information to 1. No man riding a Harley, hunting wild game, or
answer all of the following questions brandishing a body-by-Soloflex physique is
EXCEPT: above suspicion that he is on a quest to
(A) What kind of attitude does Chambers compensate for insecurities about his
have towards the political implications masculinity.
of a literary work? 2. American public opinion takes for granted
(B) What are the domains that are likely to that men strive to prove their manhood
be influenced by a political novel? through compensatory consumption, using
(C) What is the philosophical bent of Ayn whatever symbolic props are available.
Rand in her noted book Atlas 3. When public intellectuals and gender theorists
Shrugged? analyze how men pursue masculine identities
(D) Who are the literary figures that through consumption, their explanations often
influenced the writing of Joanna? parallel these popular understandings.
4. We refer to these understandings as the
Q16 Based on the passage, which of the
compensatory consumption thesis.
following sequences best represents the
flow of discussion regarding political
novels and their reception?
(A) Orwell's criticism → Resistance to Q18 In the past, researchers noticed that being born
political novels → Influence of Kafka in certain months or seasons appears to be linked
and Gogol → Dismissal of didactic to a higher risk of some diseases. In the Northern
fiction → Bellamy's influence → Hemisphere, people with multiple sclerosis tend
Historical examples to be born in the spring, while in the Southern
(B) Resistance to political novels → Hemisphere they tend to be born in November;
Influence of Kafka and Gogol → people with schizophrenia tend to have been
Bellamy's influence → Orwell's criticism born during the winter. There are numerous
→ Dismissal of didactic fiction → correlations like this, and the reasons for them
Historical examples are still foggy — a problem Tatonetti and a
(C) Orwell's categorization → Resistance to graduate assistant, Mary Boland, hope to solve
political novels → Criticism of didactic by parsing the data on a vast array of outside
fiction → Influence of Kafka and Gogol factors. Tatonetti describes it as a quest to figure
→ Bellamy's influence out “how these diseases could be dependent on
(D) Historical examples → Orwell's birth month in a way that’s not just astrology.”
categorization → Influence of Kafka Other researchers think data-mining might also
and Gogol → Resistance to political be particularly beneficial for cancer patients,
novels → Dismissal of didactic fiction → because so few types of cancer are represented
Bellamy's influence in clinical trials.
Which of the following best summarizes the
Q17 Each sentence is labeled with a number. Decide passage?
on the most logical order and enter the (A) Researchers have found that birth month
sequence of numbers in the space provided. correlations with diseases, such as multiple
Only numbers are to be entered in the space sclerosis, schizophrenia, and cancer may have
provided for the answer, and no other letters or explanations beyond astrological beliefs.
characters should be entered. (B)

Android App | iOS App | PW Website


MBA

The practice of data-mining extends beyond 2. This has led to widespread debates among
drug interactions and has the potential to policymakers, technologists, and ethicists
uncover reasons for disease correlations with about the need for transparency and
birth months and seasons. accountability in AI systems.
(C) Mary Boland and other researchers hope to 3. Moreover, biases embedded within data sets
use data-mining to solve the mystery of why can inadvertently perpetuate social
certain diseases are linked to birth months inequalities if not addressed appropriately.
and seasons, unlike astrology. 4. As AI continues to permeate various sectors,
(D) Researchers like Tatonetti and Boland are ensuring that its applications align with
focusing on using data-mining to understand societal values becomes paramount.
the reasons behind the correlation between
Q21 A sentence is missing from the given
birth months and diseases, which could also
paragraph. Read the paragraph and identify
help cancer patients.
which option out of the four sentences best fits
Q19 There is a sentence that is missing in the given in.
paragraph. Read the paragraph and identify Statement: The intermittency of renewable
which option out of the given four the sentence energy sources requires improved energy
best fits in. storage solutions.
STATEMENT: “IF MUSIC be the food of love, Passage:
play on, give me excess of it.” And if not? Well, Renewable energy technologies, such as wind
what exactly is it for? and solar power, are rapidly becoming a larger
part of our energy mix. While these sources offer
....(1).... The production and consumption of music significant environmental benefits, they also
is a big part of the economy. ....(2).....The first use come with challenges related to their
to which commercial recording, in the form of intermittent nature. .....(1)..... Unlike conventional
Edison's phonographs, was to bring music to the power plants that provide a constant energy
living rooms and picnic tables of those who could supply, renewable sources can fluctuate based
not afford to pay live musicians.....(3).... Today, on weather conditions and time of day. To
people are so surrounded by other people's address this issue, significant advancements in
music that they take it for granted, but as little as energy storage are needed to ensure a reliable
100 years ago singsongs at home, the choir in supply. .....(2)..... Modern battery technologies and
the church and fiddlers in the pub were all that other storage solutions are critical for balancing
most people heard. ....(4).... supply and demand and maintaining grid
(A) Blank 4 (B) Blank 2 stability. As we continue to integrate more
(C) Blank 3 (D) Blank 1 renewables into our energy systems, developing
and deploying effective storage technologies will
Q20 Carefully read the statements in the questions
be essential. .....(3)..... Addressing these challenges
below and arrange them in a logical order of
is vital for optimizing the use of renewable
sequence meaningfully.
energy and achieving long-term sustainability
1. With the increasing prevalence of artificial goals. .....(4).....
intelligence, questions about ethical (A) Blank 1 (B) Blank 2
considerations in algorithmic decision-making (C) Blank 3 (D) Blank 4
have become more pressing.
Q22 Out of the given statement, four statements
make a coherent paragraph. Identify the odd one

Android App | iOS App | PW Website


MBA

out and mark its number as your answer in the Q24 In the modern world, we use the language of
given space! psychology to manage and assess concepts of
agency, mastery and power. Psychology can
1. Climate risk assessment tools can help identify
provide the capacity to critique power, and
vulnerabilities within a portfolio and highlight
express suffering and frustration in the face of
potential opportunities.
oppressive economic forces the individual must
2. The key to navigating the challenges of
survive. In recent years, a powerful interest
climate change lies in building resilient
among psychologists in the ‘decolonization of
portfolios.
psychology’ itself aims to isolate and eradicate
3. Diversification across sectors and asset classes
the epistemic violence caused when Western
is crucial to mitigate the impact of climate-
models of society eclipse and obfuscate local
related disruptions.
cultural notions of ethics and identity.
4. This requires a proactive approach that
Which of the following best summarizes the
incorporates climate considerations into
main idea of the paragraph?
investment decisions.
(A) The language of psychology is used to
5. Climate-resilient portfolios prioritize working
manage concepts of agency, mastery, and
in companies and sectors positioned to thrive
power, and there is a growing movement to
in a low-carbon future.
decolonize psychology by critiquing power
and expressing frustrations against economic
Q23 Each of the following questions presents five
oppression.
statements of which four, when placed in
(B) The language of psychology is used to
appropriate order, would form a contextually
manage concepts of agency, mastery, and
complete paragraph. Pick the statement that is
power, and there is a growing movement to
not a part of that context and mark the number
decolonize psychology to prevent Western
corresponding with it in the box provided below
models from overshadowing local cultural
the question.
ethics and identity.
(C) The language of psychology is used to
1. In Judaism and Christianity, evil in human critique power, and there is a growing
conduct (which is known specifically as ‘moral movement to decolonize psychology by
evil’) is viewed as extreme defiance of God’s addressing epistemic violence caused by
commandments. Western models that overshadow local
2. The handful of academic studies that relate to cultural notions.
evil are interested in exploring the psychology (D) The language of psychology highlights
of evildoers, but the properties that guide us suffering due to oppressive economic forces,
in recognizing evil and distinguishing it from and there is a growing movement to
ordinary wrongdoing remain a puzzle. decolonize psychology by understanding and
3. For thousands of years, the concept of evil eliminating the dominance of Western models
was closely linked to a religious view of life. over local cultural ethics and identity.
4. However, despite the evident religious
connotations attached to the concept, Directions (25-30) Read the following passage
widespread use of the term has survived in and answer the given questions.
today’s secular society. A multinational company, Amit Movies, plans to
5. An act of evil radically violates that holy code. release 20 movies in the year 2025. Prabhas,
Shahrukh, Akshay, Sushant, Chris, Tom, Ryan, and

Android App | iOS App | PW Website


MBA

Sarah are selected for the lead roles. Among movies releasing in that month, which is not the
them, four are Indian actors and the remaining last month.
are American actors. No two actors will work
Q25 Which actor has the lowest releases
together in any movie, and each actor will
scheduled for 2025?
perform in at least one movie. Movies will have
(A) Shahrukh (B) Sushant
fixed release months. Indian movies are done by
(C) Akshay (D) Prabhas
only Indian actors, and so are American movies.
Further details are provided below: Q26 If Ryan's movies are not scheduled in any
1. The number of movies done by all Indian actors of the first three months, then which
is in arithmetic progression (A.P.). movie did he work on?
2. The highest number of movies done by any (A) Bahubali-1 (B) Spiderman
Indian actor and any American actor is the same, (C) Deadpool (D) Friends
and only one actor has just one movie.
3. Sarah, Ryan, Tom, and Prabhas will each do an Q27 If Prabhas and Ryan have a movie
equal number of movies. scheduled for release in the same month,
4. Chris and Shahrukh do not have any movies in then which movie is definitely not done by
the same month and have movies in different Prabhas?
months other than March and June. (A) Salaar (B) Pushpa-1
5. All the movies featuring Indian actors are (C) Spiderman (D) Both A & C
scheduled within the first nine months, with each
Q28 If the shooting of the Joker movie was
month having at least one movie.
cancelled midway, then which actor has
6. Akshay, an American actor, and Chris have the
done the highest number of movies?
same number of movies. The Indian actor with
(A) Shahrukh
two movies has the first letter of their name the
(B) Akshay
same as the Indian actor with three movies.
(C) Chris
7. Shahrukh will star in "Magadheera" (April),
(D) None of the above
"Vampire" (February), and "Avatar" (August).
8. "Bahubali 2" and "Endgame" are Indian movies Q29 In which month, the highest number of
scheduled for the same month. "Bahubali 2" is Indian movies scheduled for release?
scheduled after "Wednesday" and "Guntur" (A) July
respectively, but before "Avengers," all done by (B) May
the same actor. (C) November
9. "Joker" (November) will be released after three (D) Cannot be determined
movies done by the same person.
Q30 Which Indian actor has the highest
10. "Friends" (June) and "Pushpa 2" (March) are to
releases scheduled for 2025?
be done by the same Indian actor.
(A) Shahrukh (B) Tom
11. The groups {Salaar, Spiderman}, {Askanda,
(C) Akshay (D) Chris
Deadpool}, and {GOT, Dark Knight, Angry Man}
are each done by different actors. Directions (31-35) Read the following passage
12. The pairs (Wednesday, Salaar), (Pushpa 1 & 2), and answer the given questions.
(Magadheera, Askanda), (Guntur, GOT, Dark Mr. Mohan, while embarking on his journey from
Knight), (Friends, Angry Man), and (Joker, Mumbai to Kolkata, passes through eight other
Spiderman) are scheduled for the same month. cities named A, B, C, D, K, L, M, and N, though not
13. "Bahubali 1" and "Deadpool" are the only necessarily in that order.

Android App | iOS App | PW Website


MBA

The following table displays the distances (in The following points detail the distribution of the
kilometers) between these eight intermediate presentations:
cities:
1. Each professor presenting on these topics
K L M N
participated in at least one presentation.
A 43 63 16 26 2. For every professor presenting solely on CN,
B 14 120 41 31 there are four professors presenting on CC.
C 67 39 40 50 3. The number of professors presenting on either
D 126 20 99 109 CF or CN, but not both, is equal to the number
of professors presenting on both CF and CC.
Q31 What is the distance (in kilometers) 4. For every 10 professors presenting on CN,
between the cities K and M? there are 11 professors presenting on CF.
5. The number of professors presenting only on
Q32 What is the maximum distance (in
CF is equal to the number of professors
kilometers) between any two cities among
presenting only on CN.
the given eight cities?
6. The number of professors presenting only on
Q33 How many cities are located between K both CC and CN is one-fifth of the number
and L? presenting only on CN.
7. For every 5 professors presenting only on CN,
Q34 If K is the second-to-last intermediate city,
3 professors are presenting only on CC and CF.
which city is immediately next to Mumbai
8. The number of professors presenting only on
at the start of Mr. Mohan's journey from
both CC and CN is a prime number between
Mumbai to Kolkata?
10 and 21.
(A) City D
(B) City B Q36 What is the minimum number of
(C) City N professors presenting the presentations?
(D) None of the above
Q37 If 220 professors are presenting on
Q35 If there are no alternative routes among Climate Change, how many professors are
the eight given cities and Mr. Prateek presenting solely on Child Nurturing?
traveled 169 km from Goa to city C, what
Q38 If 195 professors are presenting on Child
is the distance (in kilometers) from Goa to
Nurturing and at least one other
city N?
presentation, how many professors are
(A) 219
presenting all three presentations?
(B) 119
(C) Either 219 or 119 Q39 What is the ratio of the number of
(D) None of the above professors presenting on Carbon
Footprints (CF) to the number of
Directions (36-40) Read the following passage
professors presenting on exactly two
and answer the given questions.
presentations?
At the Indian National Science Congress, three
(A) 20 : 17 (B) 22 : 7
social cause presentations were the most popular
(C) 5 : 7 (D) None of these
among attendees. These presentations, delivered
by professors, covered Climate Change (CC), Q40 If the number of professors presenting the
Carbon Footprints (CF), and Child Nurturing (CN). presentations is maximized, how many

Android App | iOS App | PW Website


MBA

professors are presenting in at most one Q42 How many runs did batsman E score?
presentation? (A) 100 (B) 90
(C) 80 (D) 140
Directions (41-46) Read the following passage
and answer the given questions. Q43 The batsman who scored the highest
Each of the six distinct batsmen—A, B, C, D, E, number of runs is associated with which
and F—is affiliated with a different IPL franchise: team?
Mumbai Indians (MI), Chennai Super Kings (CSK), (A) DD (B) RCB
Kolkata Knight Riders (KKR), Rajasthan Royals (C) CSK (D) None of these
(RR), Royal Challengers Bangalore (RCB), and
Q44 What is the absolute difference between
Delhi Daredevils (DD) but not in the same order.
the runs scored by batsman B and the
No two players are associated with the same
batsman from RCB?
team. In a particular IPL tournament, each
(A) 20 (B) 50
batsman scored a unique total number of runs:
(C) 40 (D) 70
80, 90, 100, 120, 140, and 150, though not in the
given order. The number of balls faced by each Q45 If all the batsmen faced different numbers
batsman was a multiple of 5, ranging from 20 to of balls, which batsman faced the second
50. The runs per ball (RPB) for each batsman highest number of balls?
were integers less than or equal to 4. (A) D (B) B
The following additional information is provided: (C) C (D) E
1. Batsman F is neither from KKR nor RCB and
Q46 How many runs did batsman A score?
scored the highest runs with the maximum
number of balls (50). Q47 The sum of all the non-negative integral solutions
2. Batsman C scored 30 runs less than the player for the given equation
from KKR.
2
(x −7x+12)
(x
2
− 5x + 5) =1
3. The difference in runs scored between
batsmen D and E is twice the difference in Q48 Arun initially covered 36 meters on a moving
runs between the batsmen from CSK and MI. travelator in the opposite direction to its motion,
Both D and E scored more runs than the followed by 48 meters in the direction of its
batsmen from CSK and MI. motion, all within 15 seconds. Subsequently, he
4. Batsman E is not associated with KKR. ran 44 meters against the travelator's motion and
5. No batsman had a lower runs/balls ratio than then 64 meters with the travelator's motion in 19
the batsman who scored 20 runs less than the seconds. Later, the travelator's speed doubled
RR batsman. due to a technical issue. What will be the time
6. Batsman B faced the fewest balls and scored Arun needs to cover 30 meters on the travelator
10 more runs than the batsman who is not against its motion and then 60 meters with its
from CSK. motion?
(A) 18 seconds (B) 21 seconds
Q41 What is the minimum number of balls (C) 24 seconds (D) 28 seconds
faced by any batsman?
(A) 30 Q49 For a 5-digit number, the sum of its digits in the ten
(B) 25 thousands, thousands, and hundreds places is 14,
the sum of its digits in the thousands, hundreds, and
(C) 20
tens places is 13, and the tens place digit is 5 less
(D) None of the above.

Android App | iOS App | PW Website


MBA

than the hundreds place digit. Find the highest resulting shape is then folded to form an open
possible 5-digit number satisfying these conditions.
cuboid of volume 108 cubic centimeters. What
was the area of the original circular sheet of
Q50 Rita, Gita, and Sita individually take different
paper?
amounts of time to complete a task, with their
(A) 36π cm
2

times being in the ratio 4:6:9. Together, they can


(B) 108π cm
2

finish the task in 5 days, working 6 hours per day.


(C) 60π cm
2

However, Rita and Gita decide to collaborate for


(D) 72π cm
2

the first 4 days, working 8:30 hours each day.


How many hours will Sita require to complete the Q56 The difference between the simple interest and
remaining task working alone? compound interest on a certain sum of money at
the same rate of interest is Rs. 60 over 2 years
Q51 If the length of a side of a regular hexagon
and Rs. 186 over 3 years. What is the principal
PQRSTU is 6 cm and V is the midpoint of ST, find
amount in rupees?
the length of PV.
−−− −−−
(A) √117 cm (B) √113 cm Q57 The average of an arithmetic progression with a
−−− −−−
(C) √111 cm (D) √108 cm non negative common difference and first term a
is 120. If the first term of the series,i.e., a is
Q52 The productivity rates of Diana, Emily, and Fiona
replaced by 9a without disturbing the other
are respectively in harmonic progression. Fiona
terms, then the new average becomes 200. How
takes three times as long as Diana to complete
many values are possible for a if a is a natural
the same amount of work. If Diana works for 5
number?
days and Emily for 8 days, Fiona will require 12
(A) 20 (B) 19
days to finish the remaining work alone. What
(C) 12 (D) 11
will be the difference between the number of
days Fiona alone would need to complete the Q58 There are five stones and each of them has a
entire job and the number of days Diana alone distinct integral weight. If two stones are
would require to complete the entire job? weighed at a time then the weights obtained in
(A) 13 days (B) 26 days kilograms are 76, 77, 79, 80, 82 , 83, 84, 85, 86
(C) 39 days (D) 18 days and 88 then what is the absolute difference
beteeen the weight of the heaviest stone and the
Q53 2x
2
− mx + 8 > 0
2
lightest stone in kilograms?
and 3x − nx + 12 ≥ 0

for all the non real values of x, then find the Q59 How many real values of x satisfy the following
minimum possible value of (3m-5n) if m,n∈ Z. equation
(A) -70 (B) -76 log
2
x +9
= 2
x 2

(C) -84 (D) -90


x −7

(A) 0 (B) 1
Q54 If
2 2
3a+2b 3 3a +2b

4a+5b
=
5
, then f ind
2
4a +3b
2
.
(C) 2 (D) 3
(A) 93
(B) 97

127 127
Q60 24 kilograms of alloy A consists of x% copper,
(C) 91
(D) 97

131 131
and 12 kilograms of alloy B consists of y% copper.
Q55 A circular sheet of paper has the largest possible A piece of z kilograms is removed from each
square cut out from it. Additionally, squares with alloy and swapped between alloys A and B. After
side lengths of 3 cm are cut out from each of the this exchange, both resulting alloys have the
four corners of the given square sheet. The same percentage of copper. Determine the value

Android App | iOS App | PW Website


MBA

of z, given that the initial copper percentages in minimum number of pens bought together by
alloys A and B are different. both Ram and Shyam given that the price of a
(A) 3 kg (B) 6 kg pencil is an integer.
(C) 8 kg (D) 9 kg (A) 5 (B) 15
(C) 18 (D) 20
Q61 A company issues access codes that are exactly 5
characters long such that repetition is allowed. Q66 If a function f(x) is defined as f(x)=
Each access code must contain at least one x
2
+ 7x + 10 , how many real roots for the
lowercase letter from e to j, at least one equation f(f(f(x))) exist?
uppercase letter from E to J, at least one digit
Q67 Ram Babu wishes to run a discount scheme at his
from 0 to 5, and at least one special character
shop, which is as follows: "Buy 3 shirts, get X
from &, *, @, $,# , or !. How many unique access
shirts free!" If he wants the discount percentage
codes can be generated under these criteria?
to not exceed 50, then the number of integral
(A) 15555(5!) (B) 15552(5!)
values that X can have is
(C) 15560(5!) (D) 15550(5!)
Q68 For how many values of a will the roots of the
Q62 In the municipal elections, the number of votes
following cubic equation be in geometric
cast against Candidate A increased by 40%
progression?
compared to those cast against him in the district
3 2
x − 13x + ax + 64 = 0
elections. As a result, Candidate A lost by a
margin that was three times the margin by which
he had won in the district elections. If a total of
220,000 votes were cast each time and only two
candidates contested for both municipal and
district elections, how many votes did Candidate
A receive in the district elections?

Q63 If A is any point lying inside a right angled


isosceles triangle PQR right angled at Q. If the
shortest distance of point A from all the three

sides of the triangle is (6 − 3√2) cm, then what
is the area of the triangle in square centimeters?

Q64 Find the sum of the n terms of the series 8, 36,


96, 200,....
(A) n(n+1)(n+2)(n+3)

(B) n(n+1)(n+2)(3n+5)

(C) n(n+1)(n+2)(2n+5)

(D) n(n+1)(n+2)(2n+3)

Q65 Ram purchased a certain number of pens and


pencils. Spending an equivalent amount, Shyam
bought three times the number of pens and 8
fewer pencils. If the cost of one pencil exceeds
that of one pen by three rupees, determine the

Android App | iOS App | PW Website


MBA

Answer Key
Q1 (B) Q31 27

Q2 (A) Q32 140

Q3 (A) Q33 4

Q4 (C) Q34 (A)

Q5 (B) Q35 (C)

Q6 (B) Q36 363

Q7 (A) Q37 55

Q8 (D) Q38 143

Q9 (D) Q39 (B)

Q10 (B) Q40 285

Q11 (D) Q41 (B)

Q12 (C) Q42 (D)

Q13 (B) Q43 (A)

Q14 (C) Q44 (C)

Q15 (C) Q45 (C)

Q16 (C) Q46 80

Q17 2134 Q47 10

Q18 (D) Q48 (B)

Q19 (D) Q49 50949

Q20 1234 Q50 15

Q21 (A) Q51 (A)

Q22 5 Q52 (B)

Q23 2 Q53 (B)

Q24 (B) Q54 (A)

Q25 (B) Q55 (D)

Q26 (C) Q56 6000

Q27 (D) Q57 (D)

Q28 (C) Q58 9

Q29 (A) Q59 (B)

Q30 (D) Q60 (C)

Android App | iOS App | PW Website


MBA

Q61 (B) Q65 (D)

Q62 120000 Q66 0

Q63 18 Q67 4

Q64 (B) Q68 1

Android App | iOS App | PW Website


MBA

Hints & Solutions


Note: scan the QR code to watch video solution

Q1. Text Solution: Correct Answer: A. Individuals have the


Explanation: autonomy to choose whether to follow the
Correct Answer: B. To illustrate the complexity moral lessons from the stories or not, reflecting
and multiplicity of meanings associated with the freedom to either accumulate or avoid
the term ‘dharma’ in different contexts. karmic burdens.
Explanation: Explanation:
Option A: This option is incorrect because, while Option A: This is the correct answer because the
the passage discusses the evolution of Indian sentence "nothing is preordained in this
thought, the example of Geldner is used to pedagogic nudge" directly implies that
illustrate the varied meanings of ‘dharma’, not the individuals have the freedom to choose whether
transition from a verb-centric to a noun-centric or not to follow the moral lessons from the
understanding. stories. This freedom is central to the idea that
Option B: This is the correct answer because the people can either accumulate or avoid karmic
author uses the example of Karl Friedrich burdens based on their choices, underscoring the
Geldner's translation of the Rigveda to highlight autonomy in their moral decision-making.
the various meanings of ‘dharma’ (‘law’, ‘order’, Option B: This option is incorrect because, while
‘duty’, ‘custom’, ‘quality’, etc.). This demonstrates it acknowledges free will, it places too much
the term's complexity and the many contexts in emphasis on the concept of rebirth and carrying
which it can be understood. past deeds and knowledge. The passage’s focus
Option C: This option is incorrect because is more on the present choices available to
Geldner’s example specifically highlights the individuals rather than on the cumulative weight
multiple meanings of ‘dharma’ rather than of past lives.
exemplifying both Mimamsaka and Upanishadic Option C: This option is incorrect because it
traditions. The passage does discuss these highlights the role of the stories in providing a
traditions, but not directly in the context of framework for moral cleansing and the decision
Geldner's translation. between adhering to dharma and accumulating
Option D: This option is incorrect because the karmic muck. Although related, it doesn’t directly
passage does not emphasize linguistic precision capture the implication of the specific sentence
in the context of Geldner’s translation. The about nothing being preordained, which is more
discussion about linguistic precision and the about the freedom of choice in the moment
verbal root ‘dhatu’ is related to Bibek Debroy’s rather than a structured framework.
comments, not Geldner’s example. Option D: This option is incorrect because, while
it discusses the model for self-improvement and
Video Solution:
emphasizes choice, it leans towards a broader
interpretation of dharma and self-improvement.
The specific sentence about nothing being
preordained is more directly about the
immediate freedom of choice in moral actions
rather than a structured model for self-
improvement.
Q2. Text Solution:

Android App | iOS App | PW Website


MBA

Video Solution: abstract and philosophical concepts rather than


rigid laws.
Option B: This option incorrectly characterizes
the verb-centric view as focusing on historical
rituals and the noun-centric view as having a
fixed universal meaning. The verb-centric
interpretation is more about ritualistic actions
Q3. Text Solution: and performance, while the noun-centric view
Explanation: focuses on philosophical ideals rather than a
Option A: This set of keywords accurately fixed, universal meaning.
reflects the central themes of the passage. It Option C: This option correctly describes the
includes "puranas" and "dharma," which are distinction. The verb-centric interpretation
central to the discussion. Keywords like "moral emphasizes the role of rituals and actions as
exemplars," "ritual sacrifice," and "self- described in the Mimamsaka traditions, focusing
improvement" align with the focus on the on the performance of duties. In contrast, the
puranas’ role in illustrating moral ideals and the noun-centric interpretation, associated with the
term 'dharma'. "Philosophical significance" and Upanishadic tradition, focuses on transcendent
"narrative influence" capture the broader ideals like truth and reality, moving beyond
implications discussed in the passage. It also ritualistic practices.
mentions Karmic burden as well as the two Indian Option D: This option incorrectly describes the
thoughts in Mimamsaka & Upanishadic traditions verb-centric view as flexible and evolving, and
Option B: Misses Dharma which is the central the noun-centric view as rigidly defined. The
Keyword in the passage verb-centric interpretation emphasizes ritualistic
Option C: Fails to highlight Mimamsaka & actions, while the noun-centric interpretation
Upanishadic traditions. Less comprehensive than focuses on philosophical and transcendent
A ideals, not necessarily rigidly tied to rituals.
Option D: Fails to highlight Mimamsaka & Video Solution:
Upanishadic traditions. Less comprehensive than
A

Video Solution:

Q5. Text Solution:


Essence of the Question:
The question asks us to identify the underlying
Q4. Text Solution: belief system that leads people to dismiss

Explanation: blatantly incoherent statements as jokes. The


Option A: This option incorrectly describes the passage discusses the irrationality of human

noun-centric interpretation as treating dharma as beings, especially focusing on structural


a set of unchanging, prescriptive laws. The noun- irrationality. The author asserts that some

centric interpretation actually emphasizes irrational beliefs or intentions are so incoherent


that they become almost unintelligible, making

Android App | iOS App | PW Website


MBA

people think they are jokes. We need to pinpoint passage does not discuss this relationship. The
which connection (beliefs, intentions, desires, key issue is the coherence between intentions
actions, social expectations, or future and the necessary actions, not how intentions
predictions) the author believes leads to this align with future outcomes.
tendency. The trap of the question:
Explanation for the Correct Answer: The trap in this question lies in the distinction
The correct answer is option (B) because the between different types of rational and irrational
author argues that when people declare highly thought processes. The passage covers various
incoherent intentions or beliefs, listeners often aspects of irrationality, such as beliefs, intentions,
interpret them as jokes because they find it and actions, making it easy to misidentify the
difficult to understand how such conflicting primary connection being discussed. One might
states of mind can coexist. The passage incorrectly focus on beliefs and desires due to
specifically illustrates this with the example of the examples provided, but the central theme of
someone intending to get shoes from the mall the author’s argument is the coherence (or lack
but not intending to go to the mall, even though thereof) between intentions and actions.
they acknowledge that going to the mall is Understanding this subtle focus is crucial to
necessary to get the shoes. This situation reveals correctly answering the question.
an intrinsic connection between intentions and
Video Solution:
actions: intending to achieve a goal (getting the
shoes) logically necessitates intending the
actions required to achieve that goal (going to
the mall). Therefore, when someone claims both
intentions and actions that are structurally
incoherent, listeners struggle to take the claim
seriously and often dismiss it as a joke.
Q6. Text Solution:
Explanation for Incorrect Answers:
Essence of the Question:
Option (A) is incorrect because while beliefs and
The question tests the our understanding of the
desires are indeed connected, the passage does
nuances in the author's argument about human
not focus on this connection in the context of
irrationality, particularly distinguishing between
dismissing incoherent statements as jokes. The
substantive and structural rationality. The task is
examples provided (e.g., overcooking eggs or the
to determine which statement aligns most closely
mall scenario) are more about the practical
with the author's perspective.
coherence between what one intends to do and
Explanation for the Correct Answer:
the actions one must take, rather than the
The correct answer is option (B) because the
relationship between beliefs and desires.
author discusses both substantive and structural
Option (C) is incorrect because desires and social
notions of rationality, emphasizing that while
expectations are not the primary focus of the
humans often exhibit irrational behaviors
discussion. The author does not link the tendency
(substantive irrationality), structural irrationality
to dismiss incoherent statements as jokes to
(incoherence) is more complex and less common.
social expectations, but rather to the structural
The discussion highlights that incoherent states
coherence between intentions and the actions
of mind are often borderline unintelligible and
required to fulfill them.
rare in reality. Instead of just labeling behaviors
Option (D) is incorrect because although
as irrational, examining the structural aspect can
intentions might relate to future predictions, the

Android App | iOS App | PW Website


MBA

help understand these behaviors better. This


Video Solution:
aligns with the author's point that a deeper
examination of structural rationality can
illuminate the nature of what might initially seem
irrational.
Explanation for Incorrect Answers:
Option (A) is incorrect because the author does
not suggest that contradictory beliefs stem
Q7. Text Solution:
merely from confusion or lack of information.
Essence of the Question:
Instead, they argue that such contradictions are
The question is asking about the purpose of the
often unintelligible, implying that it's not just a
author's discussion of "Jack" in the context of the
matter of confusion but a deeper issue related to
broader discussion on irrationality. Specifically, it
the nature of belief and intention.
seeks to determine what the author aims to
Option (C) is incorrect because although the
convey through the example of Jack holding
author notes that irrationality talk has gained
contradictory beliefs about the weather.
prominence in political rhetoric, they do not
Explanation for the Correct Answer:
suggest that this is where the concept is most
The correct answer is option (A) because the
useful. The author engages in a philosophical
author uses the example of Jack to highlight the
discussion about the nature of rationality,
challenge of conceptualizing a person who
indicating that they believe philosophical
genuinely holds two directly contradictory
discourse is a crucial arena for examining these
beliefs. This scenario is intended to illustrate that
concepts.
such extreme incoherence is not just irrational
Option (D) is incorrect because the author
but almost impossible to understand or imagine.
acknowledges that humans often display
The difficulty lies in the fundamental nature of
substantive irrationality but does not claim that it
belief itself, which typically involves a degree of
is a more realistic measure. Instead, they focus on
internal consistency. By presenting this example,
the complexities of structural rationality and its
the author underscores that while humans can
importance in understanding human behavior,
exhibit irrationality, there are limits to the kinds
suggesting a nuanced approach rather than
of incoherence that are plausible or even
favoring one over the other.
conceivable.
The trap of the question:
Explanation for Incorrect Answers:
The trap in this question lies in the subtleties of
Option (B) is incorrect because the author does
the author's argument. Each answer choice has
not suggest that contradictory beliefs stem
some basis in the passage, making it easy to
merely from confusion or lack of information.
misinterpret the main point. For instance, (C)
Instead, they argue that such contradictions are
seems appealing because the author mentions
often unintelligible, implying that it's not just a
political rhetoric, but it misses the philosophical
matter of confusion but a deeper issue related to
depth of the discussion. Similarly, (D) might seem
the nature of belief and intention.
correct because the author discusses substantive
Option (C) is also incorrect. While the author
irrationality extensively, but it overlooks the
does critique the use of unrealistic scenarios, the
emphasis on structural rationality. Recognizing
main point of the Jack example is not to criticize
the author's nuanced perspective on rationality
philosophers for their methods but to show the
and coherence is key to avoiding these traps and
difficulty in imagining such incoherent states of
selecting the correct answer.

Android App | iOS App | PW Website


MBA

mind. The focus is more on the nature of belief contemporary discussion on human irrationality
and incoherence than on philosophical does not distinguish between substantive
methodology. irrationality (holding beliefs or performing
Option (D) is incorrect because the author does actions not well-supported by evidence) and
not question the validity of irrationality as a structural irrationality (logical incoherence within
concept. Instead, the discussion focuses on the one's beliefs or intentions). The author argues
nature and limits of irrationality, particularly that popular discourse tends to focus on
structural incoherence, without dismissing the examples of substantive irrationality without
concept of irrationality altogether. addressing the more complex, and often less
The trap of the question: understood, concept of structural irrationality.
The primary trap lies in distinguishing between The passage critiques this simplification,
highlighting the absurdity of holding highlighting that many perceived instances of
contradictory beliefs and emphasizing the structural irrationality (like believing in two
difficulty in even imagining such a scenario. contradictory things simultaneously) are actually
Options (C) might seem plausible because it deal borderline unintelligible.
with contradictions and irrationality, but it miss Explanation for Incorrect Answers:
the nuanced point that the author is making Option (A) is incorrect because the passage does
about the limits of our comprehension regarding not suggest that popular discourse conflates
extreme incoherence. The correct answer (A) irrationality with disagreement or undesirability.
focuses on the core issue: the fundamental Instead, the focus is on the failure to distinguish
difficulty in conceptualizing a person with such between substantive and structural irrationality.
conflicting beliefs, which goes beyond mere Option (B) is also incorrect. While this answer is
irrationality to a deeper level of cognitive close, it is not as precise as (D). The passage does
inconceivability. argue that there is a simplification, but it
specifically points out the failure to differentiate
Video Solution:
between substantive and structural irrationality
as the main shortfall, rather than just a general
lack of nuance.
Option (C) is incorrect because it misinterprets
the passage. The author does not claim that all
attempts to explain human behavior in terms of
rationality are futile. Instead, the passage
Q8. Text Solution:
critiques the specific way irrationality is
Essence of the Question:
discussed, focusing on the lack of distinction
The question is asking us to identify why the
between substantive and structural irrationality..
popular notion of human irrationality, as
The trap of the question:
discussed in the passage, is often inadequate.
The trap in this question lies in the subtle
This requires understanding the distinctions
distinctions between the different answer
made in the passage between different types of
choices. All options appear to criticize the
irrationality (substantive vs. structural) and
popular understanding of human irrationality,
recognizing where popular discourse falls short.
but only (D) precisely captures the author's main
Explanation for the Correct Answer:
point about the failure to differentiate between
The correct answer is option (D) because the
substantive and structural irrationality. Options
passage emphasizes that much of the
(A) and (B) can seem plausible because they refer

Android App | iOS App | PW Website


MBA

to simplifications and misunderstandings, but Option (A) is incorrect because while the author
they do not accurately reflect the specific is going through an existential crisis and wants to
critique made in the passage. Option (C) can be find meaning, this answer misses a key point. The
tempting because it acknowledges human author isn't just feeling lost; he has chosen a
irrationality, but it mischaracterizes the author's specific method (analytic philosophy) to try and
argument as being more extreme than it is. find answers. This answer doesn't acknowledge
The key to avoiding the trap is to focus on the the author's proactive approach.
specific critique outlined in the passage: the lack Option (B) is incorrect because while this answer
of differentiation between the two types of is true, it is too simple. The author indeed wants a
irrationality. This understanding leads directly to purposeful life in a world that often lacks
(D) as the correct answer. inherent value, but the author’s situation is more
complex. He is struggling with doubts about
Video Solution:
meaning and has chosen a philosophical path to
address these doubts. This answer doesn't
capture the full depth of his chosen method to
resolve it.
Option (C) is incorrect because this answer only
partly describes the author's feelings. While he is
aware of the potential futility of life, he is also
Q9. Text Solution:
taking steps to understand and possibly
Essence of the Question:
overcome this feeling through philosophy. This
The question is about understanding the author's
answer ignores the author's active attempt to
struggle to find meaning in life. The author feels
find meaning.
a deep need for purpose but is troubled by the
The trap of the question:
thought that life might be meaningless. This
The tricky part of this question is recognizing the
leads them to a philosophical investigation to
author's proactive approach to his existential
find answers.
crisis. It’s easy to focus just on the emotional
Explanation for the Correct Answer:
struggle and overlook the author's decision to
The correct answer is option (D) because the
use analytic philosophy to find answers. The
author's statement shows he is experiencing a
correct answer shows not only the author's
serious personal crisis. He wants his life to matter
feelings of doubt and meaninglessness but also
and is worried that nothing really does. To
his specific method of addressing these issues
address this, he decided to dive into analytic
through detailed philosophical inquiry.
philosophy, which focuses on carefully analyzing
Understanding both aspects is crucial to
language and concepts to solve philosophical
answering correctly.
problems. The author believes that by
understanding what we mean when we talk Video Solution:

about meaning and value, he might find answers


to his crisis. This approach shows the author's
dedication to resolving his dilemma through
deep, logical thinking, making this the best
answer.
Explanation for Incorrect Answers:
Q10. Text Solution:

Android App | iOS App | PW Website


MBA

Essence of the Question: repetitive and pointless. A meaningful life


The question asks how the author's view of requires a genuine contribution to something
Sisyphus's life would change if he accepted bigger, not just symbolic significance as per
Susan Wolf's definition of a meaningful life. Wolf Wolf's metaphorical use of "larger".
defines a meaningful life as one that contributes The trap of the question:
to something bigger than oneself. The author The tricky part of the question is understanding
uses the story of Sisyphus, a mythological figure what "larger than oneself" really means in Wolf's
condemned to push a boulder up a hill forever, to definition testing the ability to apply abstract
discuss this idea. We need to figure out which philosophical definitions to concrete scenarios.
option best matches how the author would It's easy to get distracted by the metaphorical or
interpret Sisyphus's life based on Wolf's symbolic meanings and miss the core idea: a
definition. meaningful life involves a real contribution to a
Explanation for the Correct Answer: bigger purpose. The question seeks to find out if
The correct answer is option (B) because Susan the author reconciles Wolf's theoretical stance
Wolf says a meaningful life involves doing with the practical example of Sisyphus, then how
something that has a bigger purpose or his view would change. The correct answer
contributes to something beyond oneself. focuses on the fact that Sisyphus's task, despite
Sisyphus's task, pushing a boulder up a hill only any symbolic interpretation, is ultimately futile
for it to roll back down, is repetitive and has no and lacks true purpose, making his life
ultimate purpose. It doesn't contribute to meaningless according to Wolf.
anything greater. According to Wolf's definition, Video Solution:
this makes Sisyphus's life meaningless because
his efforts lead to nothing significant or lasting.
Explanation for Incorrect Answers:
Option (A) is incorrect because this option
incorrectly interprets the nature of Sisyphus's
task. The key to Wolf's definition is the
contribution to something larger than oneself,
Q11. Text Solution:
which implies a lasting impact or greater
Essence of the Question:
significance. Sisyphus's task does not transcend
The question is asking about the sentiment
his individual existence in a meaningful way; just
expressed by the phrase "Bliss it must have been
being engaged in a task doesn't make it
in that dawn to be alive!" within the context of
meaningful. The task needs to have a greater
the passage. It is crucial to understand the intent
purpose. Sisyphus's task doesn't lead to anything
and the tone behind this sentiment as used by
important; it's just endless repetition.
the author in relation to the historical excitement
Option (C) is incorrect because while Wolf does
and subsequent disillusionment associated with
suggest understanding the "larger" aspect
the rise and fall of analytic philosophy.
metaphorically, the nature of Sisyphus's task
Explanation for the Correct Answer:
remains the same—futile and repetitive.
The correct answer is option (D). The phrase
Evaluating his life based on the metaphorical
"Bliss it must have been in that dawn to be alive!"
significance of the rock does not change the
is originally from William Wordsworth's poem,
inherent meaninglessness of his actions.
referring to the excitement and idealism during
Option (D) is incorrect because the physical size
the early days of the French Revolution. The
of the rock doesn't matter. The task itself is still

Android App | iOS App | PW Website


MBA

author of the passage uses this sentiment on the importance of having clear goals in
metaphorically to describe the initial excitement philosophy.
felt by the pioneers of analytic philosophy when The trap of the question:
they believed they had found a revolutionary The trap in this question lies in the nature of the
method to solve philosophical problems through sentiment and its contextual use in the passage.
linguistic analysis. The sentiment could be interpreted as praise (A)
However, the passage also conveys that this or irony (B), and the passage does touch on
initial excitement was followed by themes of clarity in philosophy (C). However, the
disillusionment. The reference to the French key to the correct answer is understanding the
Revolution's descent from equality to tyranny author's use of the sentiment to reflect the
mirrors the decline of analytic philosophy's historical arc from excitement to disillusionment,
promise. Although it started with high hopes and mirroring the trajectory of analytic philosophy.
enthusiasm, it eventually faced criticism and was This requires careful reading and comprehension
seen as overly optimistic. Thus, the author is of the passage's overall tone and message.
using the sentiment to highlight the contrast
Video Solution:
between the initial euphoria and the eventual
disappointment.
Explanation for Incorrect Answers:
Option (A) is incorrect because while the phrase
does acknowledge the excitement and perhaps
the intellectual promise of early analytic
philosophers, the overall tone of the passage is
Q12. Text Solution:
critical. It discusses how the initial excitement
Essence of the Question:
was not sustained and how analytic philosophy
The question asks what the author implies about
failed to live up to its early promises. Therefore, it
the use of a geo-linguistic criterion (based on
is not solely praising the intellectual
language and geography) for categorizing
achievements but is also noting the subsequent
philosophies in the context of analytic
disillusionment.
philosophy, specifically focusing on the last
Option (B) is also incorrect. Although there is an
paragraph. The essence is to determine the
element of irony in the passage, especially in
author's stance on the adequacy and fairness of
how the high hopes were not fully realized, the
categorizing philosophies geographically and
primary purpose of the sentiment is not to mock
linguistically.
the overconfidence of the founders. Instead, it is
Explanation for the Correct Answer:
to highlight the journey from initial excitement to
The correct answer is an option (C) because the
disillusionment. Irony is a tool used, but it is not
author criticizes the notion of associating
the main point of the sentiment.
analytic philosophy with Anglophone (English-
Option (C) is incorrect because the passage does
speaking) regions, arguing that it's unfair to non-
discuss the goals of analytic philosophy and how
English philosophers, particularly Germans. This
it aims to clarify philosophical problems through
implies a broader critique of using geographic or
linguistic analysis. However, the sentiment "Bliss
linguistic criteria to define philosophical
it must have been in that dawn to be alive!" is
traditions. The author's sarcasm and critical tone
more about the emotional and historical context
towards the idea of geo-linguistic categorization
of the excitement and subsequent
suggest a belief that such a criterion is not a fair
disappointment rather than a direct commentary

Android App | iOS App | PW Website


MBA

or accurate way to categorize philosophical


approaches. The mention of Bernard Williams'
critique of the German philosophical writing style
serves to highlight that rigorous and complex
philosophy is not confined to any one language
or region. Thus, the author’s overall stance is that
philosophical approaches should not be limited Q13. Text Solution:
or defined by geographic or linguistic In the first paragraph, the author gives some
boundaries, indicating a belief that these examples in which the political novels were
categories are too simplistic and unfair. criticized, and the last paragraph gives a political
Explanation for Incorrect Answers: novel that is respected a lot This sense is
Option (A) is incorrect because the author expressed by Option (B), which is the best option
opposes this idea. He thinks it's unfair to judge Option (A): incorrect because, while the former
philosophies based on where they come from or certainly presents a literary trend, the last
what language they're in. He criticizes the notion paragraph does not present the extension of the
that analytic philosophy should only be trend: in fact it gives an instance of an earlier
associated with English-speaking countries. time and a trend that is opposite to the one
Option (B) is also incorrect. The author does given in the first paragraph.
mention German philosophy's style as dense, but Option (B): correct as per the reasons given
he uses this to argue against categorizing above
philosophies by language or geography. His main Option (C): incorrect because the last paragraph
point is about fairness and not about the style of too talks about the same genre of political
writing being difficult. novels.
Option (D) is incorrect because the author Option (D): incorrect because the last paragraph
doesn't express a preference for English- does not summarize the main argument of the
speaking philosophies over German ones. He passage
criticizes the idea that one style or language of Thus, the best option is Option (B)
philosophy is inherently better. His main concern
Video Solution:
is the fairness of categorizing philosophies based
on where they come from..
The trap of the question:
The main trap is misunderstanding the author's
sarcastic tone. He uses sarcasm to highlight his
point that categorizing philosophies by language
or geography is limiting and unfair. Some might
mistakenly think the author prefers English- Q14. Text Solution:
speaking philosophy or criticizes German By reading the first four sentences of the third
philosophy directly, but the real issue is about paragraph, Joanna feels that political ideas are
the broader fairness and openness in how we imposed on the fiction the moment we create a
categorize philosophical ideas. Understanding character. She explicitly mentions more didactic
this critique is key to answering the question fiction is prone to dismissal. All this implies that
correctly. she expresses political ideas as a part of
character creation or development, while Morris
Video Solution:
explicitly mentions the political purpose of the

Android App | iOS App | PW Website


MBA

book. This distinction is given in Option (C), which


is the best option
Option (A): incorrect because the info given
about Morris does not imply that he did not use
irony and satire.
Option (B): incorrect because the info given
about Morns does not support the idea that Q16. Text Solution:
he wrote only about politics. Explanation:
Option (C): correct as per the reasons given
Option C is correct because it accurately
above.
follows the sequence of key points in the
Option (D): incorrect because the Professional
passage:
success is not compared in the passage.
Thus, the best option is Option C. 1. Orwell's categorization: Orwell's views on
Video Solution: Harriet Beecher Stowe’s and Ayn Rand’s
works.
2. Resistance to political novels: The current
critical climate against campaigning novels.
3. Criticism of didactic fiction: Joanna
Kavenna’s view on the dismissal of
politically charged fiction.
4. Influence of Kafka and Gogol: Kavenna’s
Q15. Text Solution:
influence and preference for irony.
For this type of question, we need to verify each
5. Bellamy's influence: The historical impact
option individually to arrive at the correct option
of Edward Bellamy’s Looking Backward.
by elimination.
Option (A): incorrect because the last sentence Option A is incorrect because it starts with
of the first paragraph indicates that Chambers Orwell's criticism and skips directly to
has a highly negative attitude towards political Bellamy’s influence without following the flow
nature of a fiction. of the passage’s progression.
Option (B): incorrect because the last but one Option B is incorrect because it places
sentence of the passage lists out suffragist Bellamy's influence too early in the sequence
movement, populist movement and government and does not maintain the proper order of
as the fields that were influenced by the political discussion.
work of Bellamy. Option D is incorrect because it begins with
Option (C): correct because, though the last historical examples and does not follow the flow
sentence of the first paragraph does mention her of discussion about the reception and criticism of
expressing a philosophical bent, we cannot infer political novels.
what that philosophical bent is.
Video Solution:
Option (D): incorrect because we can find the list
of people who influenced Joanna, in the third
paragraph
Thus, the best option is Option (C)

Video Solution:

Android App | iOS App | PW Website


MBA

birth months and seasons, and this method


could also benefit cancer patients.
Option A introduces the idea of birth month
correlations with diseases but doesn't fully
capture the broader context of data mining
and its potential applications.

Q17 Text Solution: Option B mentions the potential of data


On a close reading of the paragraph, it can be mining beyond drug interactions which is not

observed that the paragraph focuses on explicitly mentioned. It touches on disease

American public opinion about men pursuing correlations, but it lacks a specific mention of

masculine identities through consumption. the researchers' goals and the broader
Sentence 2 introduces this topic and therefore, implications.

begins the paragraph. Sentence 1 gives examples Option C correctly refers to the efforts of

of the symbolic props mentioned in sentence 2. Mary Boland and other researchers to solve

21 is a mandatory pair. "Prove their manhood" in the mystery of disease correlations with birth
sentence 2 finds a parallel with "compensate for months, but it simplifies the broader context

insecurities about his masculinity" in sentence 1. and potential applications of their research.

So, sentence 1 provides an example of the point Video Solution:


of view mentioned in 2. "These popular
understandings" in sentence 3 refer to the
examples mentioned in sentence 1. Hence,
sentence 3 follows sentence 1. 4 closes the idea
with the introduction of the term "compensatory
consumption thesis." "These understandings" in
sentence 4 refers to what is mentioned in
Q19 Text Solution:
sentence 3.
You need to determine the correct position (1, 2,
So, 2134.
3, or 4) where this statement best fits in the
Video Solution: passage.
Analysis of the Passage:

1. Blank (1):

The sentence right before the blank is not


provided, but the passage opens with a
discussion on the broader significance of
music.
Q18 Text Solution:
The first blank might be a good place to
Correct Answer: D
introduce a philosophical or rhetorical
Explanation:
question related to the purpose of music.
Option D is the correct answer, as it accurately
2. Blank (2):
highlights that researchers like Tatonetti and
Boland are using data mining to understand The sentence after this blank explains the
the reasons behind disease correlations with historical context of how commercial
recording was used to bring music to the
masses.

Android App | iOS App | PW Website


MBA

This context doesn't directly engage with Q20 Text Solution:


the philosophical or rhetorical question in Analysis and Solution:
the given statement. It seems to focus more
Identifying the Opening Sentence:
on the technological and economic aspects
Statement 1 introduces the topic of AI and
of music.
raises the issue of ethical considerations. This
3. Blank (3):
makes it a strong candidate for the opening
The sentence after this blank describes how sentence.
music was experienced 100 years ago, Building the Narrative:
contrasting it with today's accessibility. Statement 2 logically follows 1, as it continues
This again does not fit well with the the discussion by mentioning the debates
rhetorical question posed by the statement. among various stakeholders that the ethical
concerns in A have led to. The use of "This has
4. Blank (4):
led to..." directly connects 2 to 1.
The sentence before this blank discusses Continuing the Flow:
how common music has become today, Statement 3 adds further depth to the
almost taken for granted. discussion by introducing the specific concern
The transition from this idea to another of biases in data sets, which directly relates to
related to the purpose of music might seem the ethical considerations mentioned in 1 and
abrupt. 2.
Conclusion: Concluding the Paragraph:
Statement 4 serves as a suitable conclusion by
Best Fit: Blank (1)
summarizing the overarching challenge of
The statement introduces a philosophical ensuring AI aligns with societal values, tying
query about the purpose of music, which together the concerns and discussions
sets the stage for the following discussion mentioned in the previous sentences.
on the economic, technological, and social
Correct Order: 1-2-3-4
roles of music.
Blank (1) is at the beginning of the passage, Video Solution:
making it the best place to introduce a
reflective or rhetorical question that frames
the rest of the discussion.

Answer:

The correct position for the statement is Blank


(1). Q21 Text Solution:
Video Solution: Correct Placement
Blank 1: The intermittency of renewable energy
sources requires improved energy storage
solutions.
Solution:
Blank 1:

The statement fits best in Blank 1 because it


directly addresses the challenge introduced in

Android App | iOS App | PW Website


MBA

the previous sentences regarding the Placement in Passage: "Modern battery


intermittent nature of renewable energy technologies and other storage solutions are
sources. The passage discusses the fluctuation critical for balancing supply and demand and
of renewable sources and the need for maintaining grid stability."
improvements in energy storage to manage Explanation: This blank focuses on the
these fluctuations effectively. importance of modern storage solutions in
balancing supply and demand but doesn’t
Explanation:
explicitly link this to the intermittency of
Blank 1 is appropriate because it links the renewable energy sources. It’s more about the
challenge of intermittency (highlighted in the role of storage solutions rather than the
passage) directly to the necessity for better specific need arising from intermittency.
energy storage solutions. It introduces the
Blank 4:
specific issue that is elaborated upon in
subsequent parts of the passage, thereby Placement in Passage: "Addressing these
providing a clear context for the need for challenges is vital for optimizing the use of
storage solutions. renewable energy and achieving long-term
sustainability goals."
Placement in Passage Context:
Explanation: Blank 4 discusses the importance
Blank 1: "While these sources offer significant of addressing challenges in a broader sense
environmental benefits, they also come with and highlights the goal of optimizing
challenges related to their intermittent nature. renewable energy use and achieving
The intermittency of renewable energy sustainability. It doesn’t specify that the
sources requires improved energy storage particular challenge being addressed is the
solutions." intermittency of renewable energy sources,
Inserting the statement into Blank 1 ensures that making it less precise for placing the
the passage smoothly transitions from describing statement about the need for improved
the general challenges of renewable energy to storage solutions.
emphasizing the specific need for improved Summary
storage solutions.
Blank 1 is the best placement because it
Blank 2:
directly introduces the challenge of
Placement in Passage: "To address this issue, intermittency, which is the cause for needing
significant advancements in energy storage improved storage solutions.
are needed to ensure a reliable supply." Blanks 2, 3, and 4 are less suitable as they
Explanation: While Blank 2 mentions the need either focus on the role of storage solutions
for advancements in energy storage, it doesn’t without linking them directly to intermittency
directly state the cause of this need. The or discuss the broader context of addressing
statement "The intermittency of renewable challenges without specifying the exact need
energy sources requires improved energy for improved storage.
storage solutions" directly addresses the
cause (intermittency) and the need (improved
storage), making it more appropriate for the
Video Solution:
initial explanation in Blank 1.

Blank 3:

Android App | iOS App | PW Website


MBA

Q22 Text Solution: Q23 Text Solution:


Essence: This question requires you to
understand the logical flow in arranging Answer: (2)
statements to form a meaningful paragraph. It Explanation:
emphasizes the ability to select the odd
Statement 3: Begins the paragraph by stating
statement that does not contribute to the
that for several thousand years, the concept
narrative.
of evil was closely linked to a religious view of
Hint: Focus on the theme of the paragraph and
life.
identify any statement that is irrelevant to the
Statement 1: Corroborates Statement 3 by
broader context, or does not fit within this
explaining that in Judaism and Christianity,
specific narrative flow.
evil is viewed as defiance of God's
Odd One Out: Statement 5
commandments.
Statement 5 discusses climate-resilient profiles
Statement 5: Further complements
focusing on companies and sectors thriving in a
Statements 3 and 1 by stating that an act of
low-carbon future. While relevant, it infers a
evil violates the holy code.
different aspect of climate resilience. This is not
Statement 4: Presents a contrasting idea,
directly connected to the logical flow of building
noting that despite the religious connotations,
resilient portfolios through proactive investment
the term "evil" is used extensively in today's
decisions and diversification.
secular society.
Statement 2 introduces the main idea of the
paragraph, which is about navigating the Thus, Statements 3, 1, 5, and 4 form a logical
challenges of climate change through resilient sequence explaining the historical link of the
portfolios. concept of evil to religion and its continued use
Statement 4 follows by explaining that building in secular society. Statement 2 conveys a
resilient portfolios requires a proactive approach different idea about academic studies on evil. It
that includes climate considerations in does not fit into this sequence.
investment decisions.
Video Solution:
Statement 1 fits here by detailing a specific
method for incorporating climate considerations
into investment decisions, namely through
climate risk assessment tools.
Statement 3 adds another layer to the proactive
approach by emphasizing the importance of
diversification to mitigate climate-related
Q24 Text Solution:
disruptions. It logically follows the identification
Answer Key: B. Correct. This option accurately
of vulnerabilities and opportunities.
captures the main idea by highlighting the dual
Video Solution:
role of psychology in managing concepts of

Android App | iOS App | PW Website


MBA

agency, mastery, and power, and the movement


to decolonize psychology to prevent Western
models from eclipsing local cultural ethics and
identity.
Explanation of Each Answer Option:
Option A:

Explanation: This option accurately describes Q25. Text Solution:


the role of psychology in critiquing power and From Statements 1 and 2:
expressing frustrations against economic The only possibility for the number of movies
oppression. However, it lacks the specific focus done by all Indian actors individually is 1, 2, 3,
on preventing Western models from and 4.
overshadowing local cultural ethics and The other sets of A.P. are 1, 3, 5, and 7 or 2, 3, 4,
identity, which is central to the decolonization and 5
movement described in the paragraph. But it is given that the highest number of movies
done by any Indian actor and any American actor
Option B:
is the same, which is not possible for sets of A.P. 1,
Explanation: This option correctly identifies 3, 5, and 7 or 2, 3, 4, and 5.
the main idea, which includes both the use of As only one actor has just one movie.
psychology to manage concepts of agency, Therefore, the remaining three American actors
mastery, and power, and the movement to did two movies each.
decolonize psychology to prevent Western From Statements 3 and 6:
models from overshadowing local cultural Sarah, Ryan, Tom, and Prabhas will each do an
ethics and identity. It effectively captures the equal number of movies.
dual aspects presented in the paragraph. Akshay, an American actor, and Chris have the
same number of movies
Option C:
Hence, Sarah, Ryan, Tom, and Prabhas did two
Explanation: This option focuses on movies each.
addressing epistemic violence caused by Therefore, Akshay and Chris did four movies each
Western models but does not sufficiently and Chris is an Indian actor.
emphasize the broader role of psychology in
No. of
managing concepts of agency, mastery, and Actor Nationality Movies
Movies
power. It narrows the focus too much on the
Prabhas 2
specific aspect of epistemic violence.
Shahrukh 1 or 3 Indian
Option D:
Akshay 4 American
Explanation: This option describes the Sushant 3 or 1 Indian
movement to understand and eliminate the Chris 4 Indian
dominance of Western models but does not Tom 2
sufficiently emphasize the broader role of
Ryan 2
psychology in managing concepts of agency,
Sarah 2
mastery, and power. It lacks the
From Statements 6 and 7:
comprehensive view presented in the
The Indian actor with two movies has the first
paragraph.
letter of their name the same as the Indian actor
Video Solution:

Android App | iOS App | PW Website


MBA

with three movies. scheduled within the first nine months, with each
Shahrukh will star in "Magadheera" (April), month having at least one movie.
"Vampire" (February), and "Avatar" (August). Chris, an Indian actor, did four movies. The only
It is clear that Shahrukh did three movies, possibility for the month of Chris's movie release
therefore, Sushant did only one movie. is January, May, July, and September.
Therefore, Sarah is an Indian actor, and the No. of
Actor Nationality Movies
remaining are American actors. Movies
No. of Prabhas 2 American
Actor Nationality Movies
Movies "Magadheera"
Prabhas 2 American (April),
"Magadheera" "Vampire"
Shahrukh 3 Indian
(April), (February),
"Vampire" and "Avatar"
Shahrukh 3 Indian
(February), (August)
and "Avatar" Akshay 4 American
(August) Sushant 1 Indian
Akshay 4 American January, May,
Sushant 1 Indian Chris 4 Indian July, and
Chris 4 Indian September.
Tom 2 American Tom 2 American
Ryan 2 American Ryan 2 American
Sarah 2 Indian Sarah 2 Indian
Movie Calander: Movie Calander:
Month Movie and Actor. Month Movie and Actor.
January January (Chris)
February Vampire (Shahrukh) February Vampire (Shahrukh)
March March
April Magadheera (Shahrukh) April Magadheera (Shahrukh)
May May (Chris)
June June
July July (Chris)
August Avatar (Shahrukh) August Avatar (Shahrukh)
September September (Chris)
October October
November November
December December
From Statements 4 and 5: From Statements 8 and 10:
Chris and Shahrukh do not have any movies in "Bahubali 2" and "Endgame" are Indian movies
the same month and have movies in different scheduled for the same month. "Bahubali 2" is
months other than March and June. scheduled after "Wednesday" and "Guntur"
All the movies featuring Indian actors are respectively, but before "Avengers," all done by

Android App | iOS App | PW Website


MBA

the same actor. September Avengers (Chris)


"Friends" (June) and "Pushpa 2" (March) are to be October
done by the same Indian actor.
November
The only possibility is "Bahubali 2", "Wednesday",
December
"Guntur", and "Avengers" all done by Chris.
From Statements 9, 10, and 11:
So, the only possibility for the release is
"Joker" (November) will be released after three
"Bahubali 2" in July, "Wednesday" in January,
movies done by the same person.
"Guntur" in May, and "Avengers" in September.
"Friends" (June) and "Pushpa 2" (March) are to be
No. of
Actor Nationality Movies done by the same Indian actor.
Movies
The groups {Salaar, Spiderman}, {Askanda,
Prabhas 2 American
Deadpool}, and {GOT, Dark Knight, Angry Man}
"Magadheera" are each done by different actors.
(April), Two actors did four movies, and all four movies
"Vampire" done by Chris are known, hence, Joker is the
Shahrukh 3 Indian
(February), fourth movie of Akshay.
and "Avatar" Only Sarah is an Indian actor who did two
(August) movies, hence "Friends" (June) and "Pushpa 2"
Akshay 4 American (March) are to be done by Sarah.
Sushant 1 Indian Therefore, GOT, Dark Knight, and Angry Man are
"Wednesday" done by Akshay.
(January), No. of
Actor Nationality Movies
"Guntur" Movies
(May), Prabhas 2 American
Chris 4 Indian
"Bahubali 2" "Magadheera"
(July), and (April),
"Avengers" "Vampire"
Shahrukh 3 Indian
(September). (February),
Tom 2 American and "Avatar"
Ryan 2 American (August)
Sarah 2 Indian "Joker"
Movie Calander: (November),
Month Movie and Actor. Akshay 4 American GOT, Dark
Knight, Angry
January Wednesday (Chris)
Man
February Vampire (Shahrukh)
Sushant 1 Indian
March Pushpa 2
"Wednesday"
April Magadheera (Shahrukh)
(January),
May Guntur (Chris)
"Guntur"
June Friends (May),
Chris 4 Indian
Bahubali-2 (Chris), "Bahubali 2"
July
Endgame (July), and
August Avatar (Shahrukh) "Avengers"
(September).

Android App | iOS App | PW Website


MBA

Tom 2 American Sushant 1 Indian


Ryan 2 American "Wednesday"
"Pushpa 2" (January),
(March), and "Guntur"
Sarah 2 Indian
"Friends" (May),
Chris 4 Indian
(June) "Bahubali 2"
Movie Calander: (July), and

Month Movie and Actor. "Avengers"


(September).
January Wednesday (Chris)
Tom 2 American
February Vampire (Shahrukh)
Ryan 2 American
March Pushpa 2 (Sarah)
"Pushpa 2"
April Magadheera (Shahrukh)
(March), and
May Guntur (Chris) Sarah 2 Indian
"Friends"
June Friends (Sarah)
(June)
Bahubali-2 (Chris),
July Movie Calander:
Endgame
Month Movie and Actor.
August Avatar (Shahrukh)
Wednesday (Chris),
September Avengers (Chris) January
Salaar
October February Vampire (Shahrukh)
November Joker (Akshay) Pushpa 2 (Sarah),
March
December Pushpa 1
From Statement 12: Magadheera
April
The pairs (Wednesday, Salaar), (Pushpa 1 & 2), (Shahrukh), Askanda
(Magadheera, Askanda), (Guntur, GOT, Dark Guntur (Chris), GOT
Knight), (Friends, Angry Man), and (Joker,
May (Akshay), Dark Knight
Spiderman) are scheduled for the same month.
(Akshay)
No. of
Actor Nationality Movies Friends (Sarah), Angry
Movies June
Man (Akshay)
Prabhas 2 American
Bahubali-2 (Chris),
"Magadheera" July
Endgame
(April),
August Avatar (Shahrukh)
"Vampire"
Shahrukh 3 Indian September Avengers (Chris)
(February),
October
and "Avatar"
(August) Joker (Akshay),
November
Spiderman
"Joker"
(November), December
GOT (May), From Statements 13 and 8:
Akshay 4 American
Dark Knight "Bahubali 1" and "Deadpool" are the only movies
(May), Angry releasing in that month, which is not the last
Man (June) month.

Android App | iOS App | PW Website


MBA

Hence, "Bahubali 1" and "Deadpool" are releasing Pushpa 2 (Sarah),


March
in October. Pushpa 1
"Bahubali 2" and "Endgame" are Indian movies Magadheera
April
scheduled for the same month. (Shahrukh), Askanda
"Endgame" is done by Sushant is the only
Guntur (Chris), GOT
possibility.
May (Akshay), Dark Knight
No. of (Akshay)
Actor Nationality Movies
Movies
Friends (Sarah), Angry
Prabhas 2 American June
Man (Akshay)
"Magadheera" Bahubali-2 (Chris),
(April), July
Endgame (Sushant)
"Vampire"
Shahrukh 3 Indian August Avatar (Shahrukh)
(February),
September Avengers (Chris)
and "Avatar"
October Bahubali 1, Deadpool
(August)
Joker (Akshay),
"Joker" November
Spiderman
(November),
GOT (May), December
Akshay 4 American From statement 11:
Dark Knight
(May), Angry The groups {Salaar, Spiderman}, {Askanda,
Man (June) Deadpool}, and {GOT, Dark Knight, Angry Man}
are each done by different actors.
"Endgame"
Sushant 1 Indian Therefore, Bahubali-1 and Pushpa-1 are done by
(July)
the same actor
"Wednesday"
The possibilities for these movies for actors are:
(January),
No. of
"Guntur" Actor Nationality Movies
Movies
(May),
Chris 4 Indian Salaar
"Bahubali 2"
(July), and (January),
"Avengers" Spiderman
(September). (November)
or Askanda
Tom 2 American
(April),
Ryan 2 American Prabhas 2 American
Deadpool
"Pushpa 2"
(October) or
(March), and
Sarah 2 Indian Bahubali-1
"Friends"
(October),
(June)
Pushpa-1
Movie Calander:
(March)
Month Movie and Actor.
Shahrukh 3 Indian "Magadheera"
Wednesday (Chris), (April),
January
Salaar "Vampire"
February Vampire (Shahrukh) (February),

Android App | iOS App | PW Website


MBA

and "Avatar" "Pushpa 2"


(August) (March), and
Sarah 2 Indian
"Joker" "Friends"
(November), (June)
GOT (May), Movie Calander:
Akshay 4 American
Dark Knight Month Movie and Actor.
(May), Angry Wednesday (Chris),
Man (June) January
Salaar
"Endgame" February Vampire (Shahrukh)
Sushant 1 Indian
(July)
Pushpa 2 (Sarah),
"Wednesday" March
Pushpa 1
(January),
Magadheera
"Guntur" April
(Shahrukh), Askanda
(May),
Chris 4 Indian Guntur (Chris), GOT
"Bahubali 2"
May (Akshay), Dark Knight
(July), and
(Akshay)
"Avengers"
Friends (Sarah), Angry
(September). June
Man (Akshay)
Salaar
Bahubali-2 (Chris),
(January), July
Endgame (Sushant)
Spiderman
(November) August Avatar (Shahrukh)

or Askanda September Avengers (Chris)


(April), October Bahubali 1, Deadpool
Tom 2 American
Deadpool Joker (Akshay),
November
(October) or Spiderman
Bahubali-1 December
(October), Sushant has the lowest releases scheduled for
Pushpa-1 2025.
(March)
Video Solution:
Salaar
(January),
Spiderman
(November)
or Askanda
(April),
Ryan 2 American
Deadpool
(October) or Q26. Text Solution:
Bahubali-1 No. of
Actor Nationality Movies
(October), Movies
Pushpa-1 Prabhas 2 American Salaar
(March) (January),
Spiderman

Android App | iOS App | PW Website


MBA

(November) Salaar
or Askanda (January),
(April), Spiderman
Deadpool (November)
(October) or or Askanda
Bahubali-1 (April),
Ryan 2 American
(October), Deadpool
Pushpa-1 (October) or
(March) Bahubali-1
"Magadheera" (October),
(April), Pushpa-1
"Vampire" (March)
Shahrukh 3 Indian
(February), "Pushpa 2"
and "Avatar" (March), and
Sarah 2 Indian
(August) "Friends"
"Joker" (June)
(November), Movie Calander:
GOT (May), Month Movie and Actor.
Akshay 4 American
Dark Knight Wednesday (Chris),
(May), Angry January
Salaar
Man (June)
February Vampire (Shahrukh)
"Endgame"
Sushant 1 Indian Pushpa 2 (Sarah),
(July) March
Pushpa 1
"Wednesday"
Magadheera
(January), April
(Shahrukh), Askanda
"Guntur"
Guntur (Chris), GOT
(May),
Chris 4 Indian May (Akshay), Dark Knight
"Bahubali 2"
(Akshay)
(July), and
Friends (Sarah), Angry
"Avengers" June
Man (Akshay)
(September).
Bahubali-2 (Chris),
Salaar July
Endgame (Sushant)
(January),
Spiderman August Avatar (Shahrukh)

(November) September Avengers (Chris)


or Askanda October Bahubali 1, Deadpool
(April), Joker (Akshay),
Tom 2 American November
Deadpool Spiderman
(October) or December
Bahubali-1 If Ryan's movies are not scheduled in any of the
(October), first three months, then Ryan will work on
Pushpa-1 Askanda (April) and Deadpool (October).
(March)
Video Solution:

Android App | iOS App | PW Website


MBA

Salaar
(January),
Spiderman
(November)
or Askanda
(April),
Tom 2 American
Q27. Text Solution: Deadpool

No. of (October) or
Actor Nationality Movies Bahubali-1
Movies
(October),
Salaar
Pushpa-1
(January),
(March)
Spiderman
(November) Salaar

or Askanda (January),
(April), Spiderman
Prabhas 2 American (November)
Deadpool
(October) or or Askanda

Bahubali-1 (April),
Ryan 2 American
(October), Deadpool

Pushpa-1 (October) or

(March) Bahubali-1
(October),
"Magadheera"
Pushpa-1
(April),
(March)
"Vampire"
Shahrukh 3 Indian "Pushpa 2"
(February),
and "Avatar" (March), and
Sarah 2 Indian
(August) "Friends"
(June)
"Joker"
Movie Calander:
(November),
GOT (May), Month Movie and Actor.
Akshay 4 American
Dark Knight Wednesday (Chris),
January
(May), Angry Salaar
Man (June) February Vampire (Shahrukh)
"Endgame" Pushpa 2 (Sarah),
Sushant 1 Indian March
(July) Pushpa 1
"Wednesday" Magadheera
April
(January), (Shahrukh), Askanda
"Guntur" Guntur (Chris), GOT
(May), May (Akshay), Dark Knight
Chris 4 Indian
"Bahubali 2" (Akshay)
(July), and Friends (Sarah), Angry
"Avengers" June
Man (Akshay)
(September).

Android App | iOS App | PW Website


MBA

Bahubali-2 (Chris), "Joker"


July
Endgame (Sushant) (November),
August Avatar (Shahrukh) GOT (May),
Akshay 4 American
September Avengers (Chris) Dark Knight
(May), Angry
October Bahubali 1, Deadpool
Man (June)
Joker (Akshay),
November "Endgame"
Spiderman Sushant 1 Indian
(July)
December
"Wednesday"
Prabhas and Rayan can do Askanda (April),
(January),
Deadpool (October) or Bahubali-1 (October),
"Guntur"
Pushpa-1 (March) in any order.
(May),
This means Salaar (January) and Spiderman Chris 4 Indian
"Bahubali 2"
(November) are definitely not done by Prabhas
(July), and
Video Solution: "Avengers"
(September).
Salaar
(January),
Spiderman
(November)
or Askanda
Q28. Text Solution: (April),
Tom 2 American
No. of Deadpool
Actor Nationality Movies
Movies (October) or
Salaar Bahubali-1
(January), (October),
Spiderman Pushpa-1
(November) (March)
or Askanda Salaar
(April), (January),
Prabhas 2 American
Deadpool Spiderman
(October) or (November)
Bahubali-1 or Askanda
(October), (April),
Ryan 2 American
Pushpa-1 Deadpool
(March) (October) or
"Magadheera" Bahubali-1
(April), (October),
"Vampire" Pushpa-1
Shahrukh 3 Indian
(February), (March)
and "Avatar" Sarah 2 Indian "Pushpa 2"
(August) (March), and

Android App | iOS App | PW Website


MBA

"Friends" Salaar
(June) (January),
Movie Calander: Spiderman
Month Movie and Actor. (November)
Wednesday (Chris), or Askanda
January (April),
Salaar Prabhas 2 American
Deadpool
February Vampire (Shahrukh)
(October) or
Pushpa 2 (Sarah),
March Bahubali-1
Pushpa 1
(October),
Magadheera
April Pushpa-1
(Shahrukh), Askanda
(March)
Guntur (Chris), GOT
"Magadheera"
May (Akshay), Dark Knight
(April),
(Akshay)
"Vampire"
Friends (Sarah), Angry Shahrukh 3 Indian
June (February),
Man (Akshay)
and "Avatar"
Bahubali-2 (Chris), (August)
July
Endgame (Sushant)
"Joker"
August Avatar (Shahrukh) (November),
September Avengers (Chris) GOT (May),
Akshay 4 American
October Bahubali 1, Deadpool Dark Knight
Joker (Akshay), (May), Angry
November
Spiderman Man (June)

December "Endgame"
Sushant 1 Indian
"Joker" is to be done by Akshay and he has four (July)
movies scheduled in 2025. If "Joker" movie is "Wednesday"
cancelled midway, he has three movies (January),
scheduled. "Guntur"
Therefore, Chris has the highest number of movie (May),
Chris 4 Indian
scheduled now i.e four "Bahubali 2"
(July), and
Video Solution:
"Avengers"
(September).
Tom 2 American Salaar
(January),
Spiderman
(November)
or Askanda
Q29. Text Solution:
(April),
No. of
Actor Nationality Movies Deadpool
Movies
(October) or
Bahubali-1

Android App | iOS App | PW Website


MBA

(October), The highest number of Indian movies are


Pushpa-1 scheduled in July i.e. 2.
(March)
Video Solution:
Salaar
(January),
Spiderman
(November)
or Askanda
(April),
Ryan 2 American
Deadpool
Q30. Text Solution:
(October) or
No. of
Bahubali-1 Actor Nationality Movies
Movies
(October),
Salaar
Pushpa-1
(January),
(March)
Spiderman
"Pushpa 2"
(November)
(March), and
Sarah 2 Indian or Askanda
"Friends"
(April),
(June) Prabhas 2 American
Deadpool
Movie Calander:
(October) or
Month Movie and Actor.
Bahubali-1
Wednesday (Chris), (October),
January
Salaar Pushpa-1
February Vampire (Shahrukh) (March)
Pushpa 2 (Sarah), "Magadheera"
March
Pushpa 1 (April),
Magadheera "Vampire"
April Shahrukh 3 Indian
(Shahrukh), Askanda (February),
Guntur (Chris), GOT and "Avatar"
May (Akshay), Dark Knight (August)
(Akshay) "Joker"
Friends (Sarah), Angry (November),
June
Man (Akshay) GOT (May),
Akshay 4 American
Bahubali-2 (Chris), Dark Knight
July (May), Angry
Endgame (Sushant)
Man (June)
August Avatar (Shahrukh)
"Endgame"
September Avengers (Chris) Sushant 1 Indian
(July)
October Bahubali 1, Deadpool
Chris 4 Indian "Wednesday"
Joker (Akshay),
November (January),
Spiderman
"Guntur"
December
(May),

Android App | iOS App | PW Website


MBA

"Bahubali 2" (Akshay)


(July), and Friends (Sarah), Angry
June
"Avengers" Man (Akshay)
(September). Bahubali-2 (Chris),
July
Salaar Endgame (Sushant)
(January), August Avatar (Shahrukh)
Spiderman
September Avengers (Chris)
(November)
October Bahubali 1, Deadpool
or Askanda
Joker (Akshay),
(April), November
Tom 2 American Spiderman
Deadpool
(October) or December

Bahubali-1 Chris is the Indian actor who has the highest


(October), releases scheduled for 2025.
Pushpa-1 Q31. Text Solution:
(March) Based on the table, we see that the distance
Salaar between A and K is 43 km, and between B and K
(January), is 14 km.
Spiderman Therefore, B could either be the city between A
(November) and K, or K could be the city between A and B.
or Askanda If B is the city between A and K:
(April),
Ryan 2 American The distance from A to B = Distance (A to K) -
Deadpool
Distance (B to K) = 43 - 14 = 29 km
(October) or
Bahubali-1 If K is the city between A and B:
(October), The distance from A to B = Distance (A to K) +
Pushpa-1 Distance (B to K) = 43 + 14 = 57 km
(March)
Similarly, when comparing the distances A to L
"Pushpa 2"
(63 km) and B to L (120 km), A could be between
(March), and
Sarah 2 Indian B and L, or L could be between A and B.
"Friends"
If A is the city between B and L:
(June)
Movie Calander: The distance from A to B = Distance (B to L) -

Month Movie and Actor. Distance (A to L) = 120 - 63 = 57 km

Wednesday (Chris), If L is the city between A and B:


January
Salaar
The distance from A to B = Distance (A to L) +
February Vampire (Shahrukh) Distance (B to L) = 63 + 120 = 183 km
Pushpa 2 (Sarah),
March From these comparisons, it is clear that the
Pushpa 1
distance between A and B is 57 km.
Magadheera
April From the table, we know the distances:
(Shahrukh), Askanda
May Guntur (Chris), GOT B to K = 14 km

(Akshay), Dark Knight A to M = 16 km

Android App | iOS App | PW Website


MBA

K to M = Distance (A to K) - Distance (A to M) = From these comparisons, it is clear that the


43 - 16 = 27 km distance between A and C is 24 km.
K to N = Distance (A to K) - Distance (A to N) = Therefore,
43 - 26 = 17 km
Distance from C to L = 39 km
So, N is the intermediate city between K and M. Distance from A to L = 63 km
Hence, the distance from K to N = Distance (K to Distance from A to C = 24 km
M) - Distance (M to N) = 27 - 10 = 17 km
Additionally, the table shows:
We have found that:
The distance from A to B = Distance [(A to M) + (M Distance from D to L = 20 km
to N) + (N to K) + (K to B)] = 16 + 10 + 17 + 14 = 57 Distance from C to L = 39 km
km By comparing the distance from C to D with
Thus, the cities B, K, N, M, and A are in the correct other column values, the feasible value is 59 km.
order. Thus, the distance from C to D is 59 km, and
B — 14 km — K — 17 km — N — 10 km — M — 16 since 20 + 39 = 59, L is the city between C and D.
km — A Therefore, the cities A, C, L, and D are in the
Similarly, we observe that the distance between correct order.
A and K is 43 km, while the distance between C A — 24 km — C — 39 km — L — 20 km — D
and K is 67 km. Therefore, A could either be the So, all eight cities are in the following order (in
city between C and K, or K could be the city either direction):
between A and C. B — 14 km — K — 17 km — N — 10 km — M — 16
If A is the city between C and K: km — A — 24 km — C — 39 km — L — 20 km —
The distance from A to C = Distance (C to K) - D
Distance (A to K) = 67 - 43 = 24 km The distance between the cities K and M = 17 + 10
= 27 km.
If K is the city between A and C:
Video Solution:
The distance from A to C = Distance (C to K) +
Distance (A to K) = 67 + 43 = 110 km

To further compare the distance between A and


C, we can also consider the distances from A to L
and C to L.
Similarly, we find the distances A to L (63 km)
and C to L (39 km). Thus, C could be the city Q32. Text Solution:
between A and L, or L could be the city between B — 14 km — K — 17 km — N — 10 km — M — 16
A and C. km — A — 24 km — C — 39 km — L — 20 km —
If C is the city between A and L: D
The distance from A to C = Distance (A to L) - The maximum distance between two cities at
Distance (C to L) = 63 - 39 = 24 km opposite ends, specifically from B to D, is the
longest distance among the given cities.
If L is the city between A and C:
The distance between B to D = 14 + 17 + 10 + 16 +
The distance from A to C = Distance (A to L) + 24 + 39 + 20 = 140 km.
Distance (C to L) = 63 + 39 = 102 km
Video Solution:

Android App | iOS App | PW Website


MBA

The distance from D to C = 39 + 20 = 59 km


Mr. Prateek traveled 169 km from Goa to City C
Case 1:
B — 14 km — K — 17 km — N — 10 km — M — 16
km — A — 24 km — C — 39 km — L — 20 km —
D — Goa
Q33. Text Solution: The distance from Goa to City N = 169 + 24 + 16 +

B — 14 km — K — 17 km — N — 10 km — M — 16 10 = 219 km.

km — A — 24 km — C — 39 km — L — 20 km — Case 2:

D Goa — B — 14 km — K — 17 km — N — 10 km —

N, M, A, and C are the four cities (in order) M — 16 km — A — 24 km — C — 39 km — L —

located between K and L. 20 km — D


The distance from Goa to City N = 169 – (24 + 16
Video Solution:
+ 10) = 119 km.

Video Solution:

Q34. Text Solution:


B — 14 km — K — 17 km — N — 10 km — M — 16
km — A — 24 km — C — 39 km — L — 20 km — Q36. Text Solution:

D The Venn diagram below illustrates all the

If K is the second-to-last intermediate city, Mr. categories for the three presentations: Climate

Mohan's journey starts from Mumbai to Kolkata. Change (CC), Carbon Footprints (CF), and Child
Kolkata — B — 14 km — K — 17 km — N — 10 Nurturing (CN).

km — M — 16 km — A — 24 km — C — 39 km — From Point 1: n = 0

L — 20 km — D — Mumbai
City D is immediately next to Mumbai.

Video Solution:

Q35. Text Solution:


B — 14 km — K — 17 km — N — 10 km — M — 16
km — A — 24 km — C — 39 km — L — 20 km —
D From point 2: For every professor presenting
The distance from B to C = 14 + 17 + 10 + 16 + 24 = solely on CN, there are four professors
81 km presenting on CC.

Android App | iOS App | PW Website


MBA

c : (a + d + e + g) = 1 : 4 we get [f + (c + e + g)] : [f + (b + d + g)]= 10 : 11

a + d + e + g = 4c ---(i) [f + (5e + e + 11g)] : [f + (5e + 3e + 11g)] = 10 : 11


[f + 17e] : [f + 19e] = 10 : 11
From point 3: The number of professors
presenting either on CF or CN, but not both, is So, f = 3e
equal to the number of professors presenting on Now a = b = c = 5e, d = f = 3e, g = 11e
both CF and CC.

b+d+e+c=d+g
b + e + c = g ---(ii)

From point 4: For every 10 professors presenting


on CN, there are 11 professors presenting on CF.

(c + e + f + g) : (b + d + f + g) = 10 : 11 ---(iii)

From point 5: The number of professors


presenting only on CF is equal to the number of
professors presenting only on CN.

b=c

From point 6: The number of professors


presenting only on both CC and CN is one-fifth The total number of professors = a + b + c + d + e
of the number presenting only on CN. +f+g+n
= 5e + 5e + 5e + 3e + e + 3e + 11e + 0
e:c=1:5
= 33e
c = 5e
From point 8: The number of professors
So, b = c = 5e ---(iv) presenting only on both CC and CN is a prime
From point 7: For every 5 professors presenting number between 10 and 21.
only on CN, 3 professors are presenting only on So e = 11, or 13, or 17, or 19.
CC and CF. The number of total professors will be minimum
when the value of ‘e’ is minimum which is 11.
c:d=5:3
So, total = 33e = 33 × 11 = 363.
As c = 5e; d = 3e
Video Solution:
Putting the values of b, c, and e in eq.n (ii),
we get, g = b + c + e

g = 5e + 5e + e
g = 11e

Putting the values of d, e, g in eq.n (i),


we get, a + d + e + g = 4c
Q37. Text Solution:
a + d + e + g = 4(5e)
a + 3e + e + 11e = 20e
a = 20e – 15e
a = 5e

Putting the values of c, e, g, b, d in eq.n (iii),

Android App | iOS App | PW Website


MBA

The total number of professors = 33e The number of professors presenting on Child
Thotal number professors presenting in CC = a + Nutrition and at least one other presentation = e
d+e+g + f + g = 195
= 5e + 3e + e + 11e e + 3e + 11e = 195
= 20e 15e = 195
20e = 220 e = 13
e = 11 The number of professors presenting all three
The number of professors presenting solely on presentations =g
Child Nurturing = 5e = 11e
= 5 × 11 = 11 × 13
= 55. = 143.

Video Solution: Video Solution:

Q38. Text Solution: Q39. Text Solution:

Android App | iOS App | PW Website


MBA

value of e is taken .i.e 19


The number of professors presenting in at most
one presentation
=a+b+c+n
= 5e + 5e + 5e + 0
= 15e
= 15 × 19
= 285

Video Solution:

The number of professors presenting on Carbon


Footprints (CF) = 3e + 5e + 3e + 11e
= 22e
Q41. Text Solution:
The number of professors presenting on exactly
Given:
two presentations = e + 3e + 3e
Runs scored: 80, 90, 100, 120, 140, 150
= 7e
Balls faced: 20, 25, 30, 35, 40, 45, 50
Required ratio = 22e : 7e = 22 : 7
The runs per ball (RPB) that were scored by each
Video Solution: of the batsmen was an integer which is less than
or equal to 4
Hence it could be like this: -
Runs scored Balls faced RPB
80 20/ 40 4/ 2
90 30/ 45 3/2
100 25/ 50 4/2
Q40. Text Solution: 120 30/ 40 4/ 3
140 35 4
150 50 3
Batsman F scored the highest runs with the most
number of balls.
So, F scored 150 runs from 50 balls. So, his
runs/balls are 3.
Therefore, the batsmen who scored 100 and 140
runs faced 25 and 35 balls respectively.
Runs scored Balls faced RPB Batsman
80 20/ 40 4/ 2
90 30/ 45 3/2
100 25 4
The number of professors presenting the 120 30/ 40 4/ 3
presentations is maximized, means the maximum 140 35 4

Android App | iOS App | PW Website


MBA

150 50 3 F The possible scores by D and E are any two


From points 1 and 2, we can conclude that the among 80, 120, and 140.
batsman of KKR scored 120 runs and C scored 90 For point 3, to be satisfied, the possibilities are
runs. such that the players of teams CSK and MI
Runs Balls scored 80 and 90 runs or 140 and 150 in any
RPB Batsman Franchise order so the difference will be 10.
scored faced
and D and E scored 120 and 140 runs in any
80 20/ 40 4/ 2
order, here the difference will be 20 (twice of
90 30/ 45 3/2 C
CSK and MI players).
100 25 4
But 140 and 150 for CSK and MI players are not
120 30/ 40 4/ 3 KKR possible because both D and E scored more runs
140 35 4 than the batsman from CSK or MI.
150 50 3 F So CSK and MI players scored 80 and 90 runs in
From point 6, B faced either 20 or 25 balls. any order.
If B faced 20 balls, then the score must be 80, From point 4, as E is not from KKR (120 runs), D
which contradicts point 6, as no one scored 80 – must be from KKR and scored 120 runs.
10 = 70 runs. So, E scored 140 runs and
So, B faced 25 balls and scored 100 runs, and Hence, A scored 80 runs.
100 – 10 = 90 runs scored by C is not from CSK. Runs Balls
RPB Batsman Franchise
Therefore, no batsman faced 20 balls. scored faced
Runs Balls 80 40 2 A
RPB Batsman Franchise
scored faced 90 30/ 45 3/2 C X (CSK)
80 40 2 100 25 4 B RR
90 30/ 45 3/2 C X (CSK) 120 30/ 40 4/ 3 D KKR
100 25 4 B 140 35 4 E
120 30/ 40 4/ 3 KKR 150 50 3 F
140 35 4 As C scored 90 runs, is not from CSK, C must be
150 50 3 F from MI and the player from CSK scored 80 runs.
From point 5, So, that E is from RCB, and DD is for F.
The only possibility is that the player from RR Runs Balls
RPB Batsman Franchise
must have scored 100 runs. scored faced
Runs Balls 80 40 2 A CSK
RPB Batsman Franchise
scored faced 90 30/ 45 3/2 C MI
80 40 2 100 25 4 B RR
90 30/ 45 3/2 C X (CSK) 120 30/ 40 4/ 3 D KKR
100 25 4 B RR 140 35 4 E RCB
120 30/ 40 4/ 3 KKR 150 50 3 F DD
140 35 4 The minimum number of balls faced is 25, by
150 50 3 F batsman B.
The possible scores by players CSK and MI are Video Solution:
any two among 80, 90, 140, and 150.

Android App | iOS App | PW Website


MBA

Q42. Text Solution: Q44. Text Solution:

Runs Balls Batsman Runs Balls Batsman


runs/balls Franchise runs/balls Franchise
scored faced scored faced
80 40 2 A CSK 80 40 2 A CSK
90 30/ 45 3 / 2 C MI 90 30/ 45 3 / 2 C MI
100 25 4 B RR 100 25 4 B RR
120 30/ 40 4/ 3 D KKR 120 30/ 40 4/ 3 D KKR
140 35 4 E RCB 140 35 4 E RCB
150 50 3 F DD 150 50 3 F DD
Batsman E scored 140 runs. The runs scored by B = 100
The runs scored by the batsman from RCB = 140
Video Solution:
The difference = 140 - 100 = 40 runs

Video Solution:

Q43. Text Solution:

Runs Balls Batsman Q45. Text Solution:


runs/balls Franchise
scored faced
80 40 2 A CSK Runs Balls Batsman
runs/balls Franchise
90 30/ 45 3 / 2 C MI scored faced

100 25 4 B RR 80 40 2 A CSK

120 30/ 40 4/ 3 D KKR 90 30/ 45 3 / 2 C MI

140 35 4 E RCB 100 25 4 B RR

150 50 3 F DD 120 30/ 40 4/ 3 D KKR

Batsman F, who scored the highest number of 140 35 4 E RCB


runs, is associated with the Delhi Daredevils (DD) 150 50 3 F DD
team. The second highest number of balls faced = 45.
If all the batsmen faced different numbers of
Video Solution:
balls, then
as 40 balls are faced by A, the batsman D would
have faced 30 balls.

Android App | iOS App | PW Website


MBA

That ensures 45 balls are faced by C. x


2
− 5x + 5 = 1

Batsman ⇒ x
2
− 5x + 4 = 0
Runs scored Balls faced Franchise
⇒ (x − 4)(x − 1) = 0

80 40 A CSK ⇒ x = 1 or 4
Case 3:
90 45 C MI
2
x − 5x + 5 = −1
100 25 B RR 2
⇒ x − 5x + 6 = 0
120 30 D KKR
⇒ (x − 3)(x − 2) = 0
140 35 E RCB
⇒ x = 3 or 2
150 50 F DD In case 3 chekcing whether
2
x − 7x + 12 is even or not

Video Solution: For x= 2


2
x − 7x + 12

= 4 − 14 + 12

= 2
x=3 has already been covered in the first case so
we dont need to check it.
So x has 4 possible integral values, i.e., 1, 2, 3 and
4.
Q46. Text Solution: The sum of these solutions = 1 + 2 + 3 + 4 = 10

Runs Balls Batsman Video Solution:


runs/balls Franchise
scored faced
80 40 2 A CSK
90 30/ 45 3 / 2 C MI
100 25 4 B RR
120 30/ 40 4/ 3 D KKR
140 35 4 E RCB
150 50 3 F DD Q48 Text Solution:
A scored 80 runs Let the speed of Arun be A and that of travelator
be T.
Q47 Text Solution:
Speed in direction of motion of the travelator =
For ab = 1 , there are three cases
A+T
Case 1: a0 = 1
Speed in direction opposite to the motion of the
Case 2: 1
b
= 1
travelator = A-T
Case 3: (−1)
b
= 1, when b is even
So,
Case 1: 36 48
+ = 15
2 A−T A+T
x − 7x + 12 = 0
and
⇒ (x − 3)(x − 4) = 0
44 64
+ = 19
A−T A+T
⇒ x = 3 or 4
Case 2: T aking

1 1
= X and = Y
A−T A+T

W e get

36X+48Y=15

Android App | iOS App | PW Website


MBA

and 44X+64Y=19 1 4 5 0 0-9


Solving these equations we obtain So largest such number is 50949.
1 1
X = and Y =
4 8
Video Solution:
So
A-T = 4 m/s
and A+T = 8 m/s
Solving
A= 6m/s and T = 2m/s
Due to the technical issue the speed of the
traveletor doubled
Q50 Text Solution:
So new travelator speed = 4m/s
So the time taken will be = Rita Gita Sita
30
+
60
= 21 seconds Time required
6−4 6+4 4 6 9
Ratio
Work Rate 1 1 1

Ratio 4 6 9

Video Solution:
Integral work
9 6 4
rate Ratio
Let the work rate of the workers be
Rita = 9k units/h
Gita = 6k units/h
Sita = 4kunits/h
where k is a constant.
Q49 Text Solution:
So work done in one hour by all three = 19k units
Let the number be abcde.
So total work required to finish the task =
So a+b+c = 14
5 × 6 × 19k = 570k units
and b+c+d = 13
So
Using the above equations
work done by Gita and Rita initially =
a-d = 1
15k × 8. 5 × 4 = 510k units
d = c-5
So remaining work to be done by Sita = 60k units
Since c≤9
Time required by Sita to complete the work =
60k

so d≤4 4k

=15 hours
So using the above conditions lets have a look at
the value of the digits of the number(first take Video Solution:
the values of d and then fill others accordingly, e
doesn't have any condition so any value is
possible)
b=(13-c-
a=(d+1) c=(d+5) d(≤4) e
d)
5 0 9 4 0-9
4 1 8 3 0-9 Q51 Text Solution:

3 2 7 2 0-9
2 3 6 1 0-9

Android App | iOS App | PW Website


MBA
5 8 12
+ + = 1
x 2x 3x

5+4+4
⇒ = 1
x

⇒ x = 13

Number of days required by Diana to finish the


work alone = 13 days
Number of days required by Fiona to finish the
work alone = 39 days
So all the sides are 6cm
So the difference between the two = 26 days
If we join PT, it will form a 30:120:30 triangle PUT,
so ∠UTP = 30° Video Solution:
Also since the sides of 30:120:30 triangle are in 1:

√3 :1 ratio,

PT = 6√3 cm

Also ∠PTV will be 90°


So
2 2 2
PV = PT + TV

– 2 2
Q53 Text Solution:
2
⇒ PV = (6√3 ) + 3

⇒ PV
2
= 117 For x to be non real, D<0
−−− 2
⇒ P V = √117 cm F or 2x − mx + 8 > 0
2
m − 4(2)(8) < 0

⇒ −8 < m < 8

Video Solution: F or 3x
2
− nx + 12 > 0
2
n − 4(3)(12) < 0

⇒ −12 < n < 12

So to find the minimum possible value of (3m-


5n), we should minimize m and maximize n
(3m − 5n)
min

=3(-7)-5(11)
Q52 Text Solution: =-76

If the productivity rate(efficiency) is in HP, then Video Solution:


the time taken to complete the work will be in AP.
So, let the time taken by Diana to complete the
work be x days.
So time taken by Fiona = 3x days
Time taken by Emily = =2x (arithmatic mean
x+3x

of the rest of two)


Work done per day by Diana =
1

x Q54 Text Solution:


Work done per day by Emily =
1
3a+2b 3
2x =
4a+5b 5
Work done per day by Fiona = 1

3x ⇒ 15a + 10b = 12a + 15b


If total work done is 1, then 5
⇒ a = b
3

So

Android App | iOS App | PW Website


MBA

CI in 2 years = 2i +
2 2
3a +2b r×i

2 2 100
4a +3b
2

CI in 3 years = 3i + 3( 100 ) +
r×i r ×i
2
5b 2 2
3( ) +2b 100

So
3
=
2
5b 2
4( ) +3b
3 r×i
(2i + ) − 2i = 60
2 2
100
75b +18b
= r×i
100b +27b
2 2
⇒ = 60
100

=
93
And
127
2
r×i r ×i
[3i + 3( ) + ] − 3i = 186
Video Solution: 100 100
2

2
r×i r ×i
⇒ 3( ) + = 186
100 2
100

r r×i
⇒ 180 + ( ) = 186
100 100

⇒ 0. 6r = 6

⇒ r = 10%

So

Q55 Text Solution: i =


100
× 60 = Rs 600
r

Let the side of the largest square be s cm. So

Since the squares of side 3 each are cutout from i =


P ×r

100
the four corners, it will result in the length and 100i 100×600
⇒ P = = = Rs 6000
breadth of the cuboid being (s-3-3) cm and its r 10

So
height being 3 cm.
So,
Volume of cuboid = (s-6)(s-6)(3)
⇒ 108 =
2
(s − 6) (3)
Video Solution:

2
⇒ (s − 6) = 36 ⇒ s = 0 or 12

Since the side cannot be 0 so, s =12 cm.


So Diameter of the circle= diagonal of the circle
– –
= √2 s = 12√2 cm

So radius = 6√2 cm
So area of the initial circular sheet =
2 2
Q57 Text Solution:
πr = 72π cm
Let the initial series be
a,a2 , a3 , . . . , an
Video Solution: So
a + a2 + a3 +. . . +an = 120n

and

9a + a2 + a3 +. . . +an = 200n

So subtracting the first equation from the other


we obtain
8a = 80n
⇒ a = 10n
Q56 Text Solution:
So possible values of a are = 10,20,30,...120
Let the simple interest earned in a year be i and
(Since smallest term will always be less than or
let the rate of interest be r%
equal to average)
SI in 2 years =2i and SI in 3 years = 3i

Android App | iOS App | PW Website


MBA

So checking the extreme values


If a = 120, n=12
So to make average 120
a = a2 = a3 =. . . = a12 = 120 (possible case
as the difference is non positive integer 0)
If a = 10, n=1, in this case average = 10 and not
120, so the case is not possible. Q59 Text Solution:
So checking for the next value x +9
2

log = 2
x
If a = 20, n = 2 then to obtain average as 120
2
x −7

2
x +9
(So a possible case)
2
a2 = 230 ⇒ = x
2
x −7

So total possible values of a are 20,30,40,...,120 ⇒ x


2
+ 9 = x
4
− 7x
2

So 11 possible values. 4 2
⇒ x − 8x − 9 = 0

Video Solution: ⇒ (x
2
− 9)(x
2
+ 1) = 0

2
⇒ x = 9 or − 1
2 −−

⇒ x = ±3 or ± √−1
−−

S ince ± √−1 are not real, they will not

be considered
Also base of log cannot be negative, so 3 is the
only possible value of x. So only one real value of
Q58 Text Solution:
x satisfies the equation.
So the average weight of all the stones =
76+77+79+80+82+83+84+85+86+88 Video Solution:
= 41 kg
20

(There are 10 measures with 2 stones each)


So the weight of two heaviest stones = 88
and the weight of the two lightest stones = 76
So the weight of the middle stone =
(5 × 41) − 88 − 76 = 41kg
The second heaviest weight is going to be the
highest weight plus the third highest weight Q60 Text Solution:
So letting the weight of the heaviest stone to be As the swapping results in equal percentage of
x copper in both the alloys therefore the ratio in
86 = x+41 which alloy A and alloy B are present in the
⇒ x = 45 kg resulting alloys will also be same.
Similarly the second lightest weight is going to So,
Quantity of alloy A in resultant alloy A
be the lightest weight plus the third lightest
Quantity of alloy B in resultant alloy A

weight, so taking the lightest weight to be y Quantity of alloy A in resultant alloy B


=
77 = y+41 Quantity of alloy B in resultant alloy B

y = 36 kg
24−z z
⇒ ⇒ =
z 12−z

So the difference between the two = 45-36 = 9 2 2


⇒ z − 36z + 288 = z
kg ⇒ 288 − 36z = 0

⇒ z = 8 kg

Video Solution: Video Solution:

Android App | iOS App | PW Website


MBA
2.8x−220000
= 3
220000−2x

⇒ 2. 8x − 220000 = 660000 − 6x

⇒ 8. 8x = 880000

⇒ x = 100000
So votes casted in favour of candidate A in
district elections = 220000-100000 = 120000

Q61 Text Solution: Video Solution:


So we have 5 characters to choose from 4
categories and since one from each category is
necessary
No of ways of choosing category with 2
characters = =4
4
C
1

No of ways of choosing two characters from that


category for two different characters= = 15
6
C
2 Q63 Text Solution:
No of ways of choosing two characters from that
category for two same characters = =6
6
C1

No of ways of selecting rest of the three


characters = 6C 1 ×
6
C
1
×
6
C
1
= 216
No of ways of arranging the 5 characters with
five different characters= 5!
No of ways of arranging the 5 characters with
two same characters= 5!

So total number if ways =

4(15 × 216 × 5! + 6 × 216 ×


5!

2
) =
4 × 5!(15 × 216 + 3 × 216) = 15552(5!)

Video Solution:

If PQR is a right angled isosceles triangle then it


is a 45:45:90 triangle, which means its sides are
in ratio

PQ:QR:PR=1 : 1 : √2

The point inside the circle equidistant from all the


Q62 Text Solution: three sides is the incenter of the circle, so the

Let the number of votes cast against candidate A inradius = (6 − 3√2) cm

in district elections be x. Let the sides be x, x, √2 x, so
So the number of votes cast against candidate A r=
a+b−c

in municipal elections will be 1.4x. –


⇒ 6 − 3√ 2 =
x+x−√2 x

2
Margin of victory in district elections = (220000-
2(6−3√2 )

x)-x = 220000 - 2x ⇒ x =
2−√2

Margin of loss in municipal elections = 1.4x- ⇒ x = 6 cm

(220000-1.4x) = 2.8x-220000 So area of triangle PQR = = 18 cm2


1
× 6 × 6
2

So,

Android App | iOS App | PW Website


MBA

Video Solution: ⇒ ax + bx + 3b = 3ax + bx − 8x + 3b

− 24

⇒ 8x − 2ax + 24 = 0
12
⇒ a = 4 +
x

Since both x and a should be positive integers,


possible values of a and x
x a
Q64 Text Solution: 1 16
Required sum 2 10
8+36+96+200+...n terms 3 8
= 2(4)+4(9)+6(16)+8(25)+...n terms 4 7
So general term of the series =
6 6
2 2 3
2n(n + 1) = 2n(n + 2n + 1) = 2n
12 5
+ 4n
2
+ 2n So the minimum number of pens bought = 4a =
So sum of the series = 20
3 2
∑ 2n + 4n + 2n
Video Solution:
3 2
= 2∑n + 4∑n + 2∑n

2 2
n (n+1) n(n+1)(2n+1)
= 2[ ] + 4[ ]
4 6

n(n+1)
+ 2[ ]
2

n(n+1) 2(2n+1)
= n(n + 1)[ + + 1]
2 3
Q66 Text Solution:
n(n+1)(3n +11n+10)
2
f(x)=x2 + 7x + 10
=
6
f(x)=0
n(n+1)(n+2)(3n+5)
2
= ⇒ x + 7x + 10 = 0
6

⇒ (x + 5)(x + 2) = 0
Video Solution:
⇒ x = − 5 or − 2

So f(x)=0 has two roots


Finding roots of f(f(x))=0
2
⇒ f (x + 7x + 10) = 0

2 2
⇒ x + 7x + 10 = −5 or x + 7x + 10 =

− 2
2 2
⇒ x + 7x + 15 = 0 or x + 7x + 12 = 0
Q65 Text Solution:
So first equation has no real roots and the
Let the number of pens and pencils bought by
second equation has two roots -3 and -4.
Ram be a and b respectively.
So
So the numbers of pens and pencils bought by
f (f (f (x))) = 0
Shyam will be 3a and (b-8) respectively. 2 2
⇒ x + 7x + 12 = −3 or x + 7x + 12 =
If the price of one pen is x then price of one
−4
pencil will be x+3. 2 2
⇒ x + 7x + 15 = 0 or x + 7x + 16 = 0
So
ax+b(x+3)=3ax+(b-8)(x+3)

Android App | iOS App | PW Website


MBA

Both the equations above have imaginary roots αβγ = −64

and hence f(f(f(x)))=0 has no real roots. ⇒


β
× β × βr = −64
r

Video Solution: ⇒ β
3
= −64

⇒ β = − 4

Also

α + β + γ = 13

Let 'r' be the common ratio,

1
⇒ β( + 1 + r) = 13
r

1 −13
Q67 Text Solution: ⇒
r
+ 1 + r =
4

Let the list price per shirt be Rs. 1. ⇒ r +


1
=
−17

r 4
Total marked price = total number of shirts given −1
⇒ r = − 4 or
= Rs. (3 + X) 4

So the only set of roots are 1, -4, 16.


Total discount given = total number of shirts
So
given for free = Rs. X
3 2
x − 13x + ax + 64 = (x − 1)(x + 4)(x
Now, given that
X
× 100 ≤ 50
3+X

=> X ≤ 3 − 16)

Possible values of X = {0, 1, 2, 3} (Since X is the ⇒ a = − 52

which is the only possible value of a.


number of shirts, it cannot be negative.)
Number of possible values of X = 4 (correct Video Solution:
answer)

Video Solution:

Q68 Text Solution:


Considering the roots of the equation to be
α, β and γ , we get
−(−13)
α + β + γ = = 13
1

−64
αβγ = = −64
1

Also since α, β and γ are in GP so lets consider


them as
β
α = and γ = βr
r

So

Android App | iOS App | PW Website


MBA

Android App | iOS App | PW Website

You might also like